Page 1 of 70 PTS 2020|1|Test 4 - Solutions |ForumIAS

Q.1) Ans) c Exp) The type of vegetation which has grown on its own naturally without any aid from human beings or those which have been left untouched or undisturbed by the human beings for a long time are termed as “Virgin vegetation”. The virgin vegetation, which are purely Indian are known as endemic or indigenous species but those which have come from outside India are termed as exotic plants. Thus statement 3 is correct. Example - Unreachable areas of the Amazon rainforest, Trenches in the Ocean etc. Plant community which is endemic to a particular area is known as native species. Plant community formed due to primary succession in an ecosystem are known as Pioneer species. Plant community newly introduced in an ecosystem is known as exotic species.

Source) NCERT- Class IX- SOCIAL SCIENCE CONTEMPORARY INDIA PART 1 CHAPTER 5, PAGE No. 42

Q.2) Ans) c Exp) Both the statements are correct. India is the only country in the world that has both tigers and lions in their natural habitat (in the wild).The Asiatic lion is Panthera leo leo species in India. Its range is restricted to the Gir National Park and environs in the Indian state of Gujarat. On the IUCN Red List, it is listed as Endangered because of its small population size and area of occupancy. Lions inhabit remnant forest habitats in the two hill systems of Gir and Girnar that comprise Gujarat's largest tracts of tropical and subtropical dry broadleaf forest, thorny and savanna, and provide valuable habitat for a diverse flora and fauna. Five protected areas currently exist to protect the Asiatic lion: Gir Sanctuary, Gir National Park, Pania Sanctuary, Mitiyala Sanctuary, and Girnar Sanctuary. The Gir Forest is the last remaining habitat of the Asiatic lion. Across India, six landscape complexes were surveyed that host tigers and have the potential to be connected. These landscapes comprise the following: Shivaliks-Ganjetic floodplains, Central Indian Highlands, Eastern ghats, Western ghats, Brahmaputra flood plains and the Sundarbans.

Source) NCERT- Class IX- SOCIAL SCIENCE CONTEMPORARY INDIA PART 1 CHAPTER 5, PAGE No. 48 https://en.wikipedia.org/wiki/Asiatic_lion#Distribution_and_habitat https://en.wikipedia.org/wiki/Bengal_tiger

ForumIAS Offline Guidance Centre 2nd Floor, IAPL House, 19, Pusa Road, Karol Bagh, New Delhi – 110005 | [email protected]|9821711605 Page 2 of 70 PTS 2020|1|Test 4 - Solutions |ForumIAS

Q.3) Ans) c Exp) During the south-west monsoon period after having rains for a few days, if rain fails to occur for one or more weeks, it is known as break in the monsoon. These dry spells are quite common during the rainy season. These breaks in the different regions are due to different reasons: Statement 1 is correct (i) In northern India rains are likely to fail if the rain-bearing storms are not very frequent along the monsoon trough or the ITCZ over this region. It also occurs if the ITCZ shifts northward over the foothills of Himalayas and causes heavy rainfall along the foothills of himalayas. This northward shift of the ITCZ creates a high pressure over the plains and thus the South West monsoon winds are not able to flow Statement 2 is correct (ii) Over the west coast the dry spells are associated with days when winds blow parallel to the coast. This fails to produce an orographic effect and thus leads to a break in monsoon. Such breaks are likely to occur in the month of August.

Source) Class XI NCERT, Chapter 5, page no. 40

Q.4) Ans) d Exp) 1st statement is correct. Coriolis force: An apparent force caused by the earth’s rotation. The Coriolis force is responsible for deflecting winds towards the right in the northern hemisphere and towards the left in the southern hemisphere. This is also known as ‘Ferrel’s Law’. 2nd statement is correct. Coriolis force is absent at the equator and highest at the poles. 3rd statement is correct. Where a cyclone forms, the direction it spins and the general path it takes are determined by a range of factors including the Coriolis effect caused by the Earth's rotation. Cyclones are formed due to the presence of a low-pressure zone. As the low pressure intensifies over a period of time (depending on various factors) the surrounding air rushes in to fill it. As the air is rushing in it will rotate according to the Ferrel’s law and will cause cyclones to rotate anticlockwise in the northern hemisphere and clockwise in the southern hemisphere. 4th statement is incorrect. The Coriolis effect bends the direction of surface currents to the right in the Northern Hemisphere and left in the Southern Hemisphere. This is true for the deep- water circulation as well.

KB) The Ekman spiral, named after Swedish scientist Vagn Walfrid Ekman (1874-1954) who first theorized it in 1902, is a consequence of the Coriolis effect. When surface water molecules move by the force of the wind, they, in turn, drag deeper layers of water molecules below them. Each

ForumIAS Offline Guidance Centre 2nd Floor, IAPL House, 19, Pusa Road, Karol Bagh, New Delhi – 110005 | [email protected]|9821711605 https://t.me/UPSC_PDF www.upscpdf.com

PTS 2020|1|Test 4 - Solutions |ForumIAS

layer of water molecules is moved by friction from the shallower layer, and each deeper layer moves more slowly than the layer above it, until the movement ceases at a depth of about 100 meters (330 feet). Like the surface water, however, the deeper water is deflected by the Coriolis effect—to the right in the Northern Hemisphere and to the left in the Southern Hemisphere. As a result, each successively deeper layer of water moves more slowly to the right or left, creating a spiral effect. Because the deeper layers of water move more slowly than the shallower layers, they tend to “twist around” and flow opposite to the surface current.

Source) NCERT- Class IX- SOCIAL SCIENCE CONTEMPORARY INDIA PART 1 CHAPTER 4, PAGE No 27

Q.5) Ans) a Exp) Option (a) is the correct answer. Transparency International’s 2018 Exporting Corruption report is an independent assessment of the enforcement of the Organization for Economic Co-operation and Development’s (OECD) ‘Anti-Bribery Convention’, which requires parties to criminalize bribery of foreign public officials and introduce related measures. The Convention is a key instrument for curbing global corruption because the 44 signatory countries are responsible for approximately 65 percent of world exports and more than 75 per cent of total foreign direct investment outflows. This twelfth such report assessed enforcement in China, Hong Kong Special Administrative Region of the People's Republic of China, India and Singapore, which are not parties to the OECD Convention but are major exporters, accounting for 18 percent of world exports.

KB) ● The World Economic Forum publishes a comprehensive series of reports which examine in detail the broad range of global issues it seeks to address with stakeholders as part of its mission of improving the state of the world. Besides reports on its key events and standalone publications such as the Global Competitiveness Report, the Global Risks Report and the Global Gender Gap Report, the Forum produces landmark titles covering the environment, education, individual industries and technologies. ● The Financial Action Task Force (FATF) is an inter-governmental body established in 1989 by the Ministers of its Member jurisdictions. The objectives of the FATF are to set standards and promote effective implementation of legal, regulatory and operational measures for combating money laundering, terrorist financing and other related threats to the integrity of the international financial system. The FATF is therefore a “policy-

ForumIAS Offline Guidance Centre 2nd Floor, IAPL House, 19, Pusa Road, Karol Bagh, New Delhi – 110005 | [email protected]|9821711605 https://t.me/UPSC_PDF www.upscpdf.com

PTS 2020|1|Test 4 - Solutions |ForumIAS

making body” which works to generate the necessary political will to bring about national legislative and regulatory reforms in these areas. ● TRACE membership helps companies conduct business ethically and in compliance with the U.S. Foreign Corrupt Practices Act, U.K. Bribery Act and other anti-bribery legislation.

Source) https://www.transparency.org/whatwedo/publication/exporting_corruption_2018

Q.6) Ans) b Exp) Statement 1 is incorrect. Continentality of any area leads to extreme climatic conditions. Thus, continentality cannot be a reason for winters being mild as they will only aggravate the existing climatic conditions and not balance it. Statement 2 and 3 are correct. India has Himalayas to the north, which have an average height of about 6,000 metres. The Himalayas therefore act as a barrier to cold winds coming from Tibet and other colder regions in Central Asia (Gobi Desert etc). It is because of these mountains that the subcontinent experiences comparatively milder winters as compared to central Asia. India’s vast coastline leads to a moderating influence on India’s climate and thus results in milder winters as compared to Central Asia. The Tropic of Cancer also passes through the middle of the country from the Rann of Kachchh in the west to Mizoram in the east. Almost half of the country, lying south of the Tropic of Cancer, belongs to the tropical area. All the remaining area, north of the Tropic, lies in the sub-tropics. Therefore, India’s climate has characteristics of tropical as well as subtropical climates. This also leads to milder winters in India.

Source) NCERT- Class IX- SOCIAL SCIENCE CONTEMPORARY INDIA PART 1 CHAPTER 4, PAGE No. 27 Feature)

Q.7) Ans) c Exp) Option (c) is the correct answer. Statement 1 is correct. Eastern Ghats are a discontinuous range of mountains along India's coast due to the large number of rivers dissecting the eastern ghats as they flow towards the Bay of Bengal. They run from northern Odisha through Andhra Pradesh to Tamil Nadu in the south passing some parts of Karnataka and Wayanad district of Kerala.

ForumIAS Offline Guidance Centre 2nd Floor, IAPL House, 19, Pusa Road, Karol Bagh, New Delhi – 110005 | [email protected]|9821711605 https://t.me/UPSC_PDF www.upscpdf.com

PTS 2020|1|Test 4 - Solutions |ForumIAS

Statement 2 is not correct. The western coastal plains are narrow strip of coastal plain 50 kilometers in width between the west coast and the western ghats whereas the Eastern coastal plains are much broader stretch of landmass of India, lying between eastern Ghats and the bay of bengal. Statement 3 is correct. A number of east flowing rivers, Mahanadi, Godavari, Krishna, Kaveri, Vaigai drain into the bay of bengal. This happens due to the SoutEast slope of the peninsular plateau.

KB) Other east flowing rivers include - Subarnarekha, Brahmani, Baitarani and Penneru.

Source) Ncert class 6 geography, chapter 7, page no. 51 Feature)

Q.8) Ans) c Exp) Option (c) is the correct answer. Fixed-dose combination (FDC) medicines are a cocktail of two or more active drugs packed in a single dose. Government has recently banned FDCs. Statement 1 is correct. FDCs help to improve adherence, simplify therapy and/or to maximize benefit for the patient along with the added effects of the multiple medicinal products given together. Instead of buying two, or more, separate medicines, a patient can buy just one FDC medicine to treat multiple illness symptoms, which typically works out easier on the wallet. Statement 2 is also correct. The Health Ministry has banned FDCs after the recommendation of the Drugs Technical Advisory Board that there is no therapeutic justification for the ingredients contained in the banned FDC drugs and that these medicines may involve risk to human beings. Chances of adverse drug effects and drug interactions can go up if medicines are combined instead of being taken separately. Unnecessary use of combination drugs makes the human body resistant to treatment.

KB) The raison d'etre behind FDCs is to improve adherence, simplify therapy and/or to maximise benefit for the patient courtesy the added effects of the multiple medicinal products given together. Popular FDCs, now banned, include the painkiller Saridon, the skin cream Panderm, antibiotic Lupidiclox and combination diabetes drug Gluconorm PG. A government document titled 'Policy Guidelines for Approval of Fixed Dose Combinations in India', released in June 2013 had claimed that "FDCs have shown to be particularly useful in the treatment of infectious diseases like HIV, malaria and tuberculosis where giving multiple

ForumIAS Offline Guidance Centre 2nd Floor, IAPL House, 19, Pusa Road, Karol Bagh, New Delhi – 110005 | [email protected]|9821711605 https://t.me/UPSC_PDF www.upscpdf.com

PTS 2020|1|Test 4 - Solutions |ForumIAS

antimicrobial agents is the norm. FDCs are also of use in chronic conditions especially when multiple disorders often coexist." Worryingly, published studies have long claimed that FDCs are often prescribed to cover up for diagnostic imprecision-likely making them extremely popular with quack doctors. Hence, in a bid to stop the irrational use of FDCs, the Union Health Ministry on Wednesday banned the manufacture, sale or distribution of 328 varieties of FDC drugs for human consumption. The move will likely affect over 6,000 medicine brands.

Source) https://www.businesstoday.in/sectors/pharma/what-are-fdc-drugs-and-why-has- the-govt-decided-to-ban-them/story/282350.html

Q.9) Ans) d Exp) Option (d) is the correct answer. The running water of Chambal cuts through the clayey soils and makes deep channels as gullies. The land becomes unfit for cultivation and is known as bad land. In the Chambal basin such lands are called ravines. Sometimes water flows as a sheet over large areas down a slope. In such cases the top soil is washed away. Thus such areas of are prone to sheet erosion. The Chambal River is a tributary of the Yamuna River in central India, and thus forms part of the greater Gangetic drainage system. The river flows north-northeast through Madhya Pradesh, running for a time through Rajasthan, then forming the boundary between Rajasthan and Madhya Pradesh before turning southeast to join the Yamuna in Uttar Pradesh state. Tributaries of Chambal are Banas, Mej, Parbati, Kali Sindh, Shipra It is a legendary river and finds mention in ancient scriptures. The perennial Chambal originates at janapav, south of town, near manpur , on the south slope of the Vindhya Range in Madhya Pradesh. The Chambal River hosts 2 species of crocodiles– the mugger and gharial, freshwater turtles, gangetic river dolphins.

Source) Chap-1, page no.-11, Contemporary India-2

Q.10) Ans) c Exp) Option (c) is the correct answer. Biosphere reserves are sites established by countries and recognized under UNESCO's Man and the Biosphere (MAB) Programme to promote sustainable development based on local community efforts and sound science. The programme of Biosphere Reserve was initiated by UNESCO in

ForumIAS Offline Guidance Centre 2nd Floor, IAPL House, 19, Pusa Road, Karol Bagh, New Delhi – 110005 | [email protected]|9821711605 https://t.me/UPSC_PDF www.upscpdf.com

PTS 2020|1|Test 4 - Solutions |ForumIAS

1971. The purpose of the formation of the biosphere reserve is to conserve in situ all forms of life, along with its support system, in its totality, so that it could serve as a referral system for monitoring and evaluating changes in natural ecosystems. The first biosphere reserve of the world was established in 1979, since then the network of biosphere reserves has increased to 631 in 119 countries across the world. Biosphere reserves have three interrelated zones that aim to fulfill three complementary and mutually reinforcing functions. The core area comprises a strictly protected ecosystem that contributes to the conservation of landscapes, ecosystems, species and genetic variation. The buffer zone surrounds or adjoins the core areas, and is used for activities compatible with sound ecological practices that can reinforce scientific research, monitoring, training and education. The transition area is the part of the reserve where the greatest activity is allowed, fostering economic and human development that is socio-culturally and ecologically sustainable. Biosphere reserves are areas comprising terrestrial, marine and coastal ecosystems. Total number of Biosphere reserves in India is 18. From the map the biosphere reserves can be arranged from south to north and thus option C is correct.

ForumIAS Offline Guidance Centre 2nd Floor, IAPL House, 19, Pusa Road, Karol Bagh, New Delhi – 110005 | [email protected]|9821711605 https://t.me/UPSC_PDF www.upscpdf.com

PTS 2020|1|Test 4 - Solutions |ForumIAS

Source) NCERT- Indian Physical Environment of Class 11, Chapter no 5, Page no. 64.

Q.11) Ans) a Exp) Option (a) is the correct answer. TROPICAL DECIDUOUS FORESTS These are the most widespread forests of India. They are also called ‘Monsoon forests’ and receive rainfall between 200 cm and 70 cm. Trees of this forest-type shed their leaves for about six to eight weeks in dry summers. This is a part of a natural process where the trees shed their leaves to conserve water. This process is known as Abscission. On the basis of the availability of water, these forests are further divided into moist and dry deciduous. The former is found in areas receiving rainfall between 200 and 100 cm. These forests therefore exist mostly in the eastern part of the country – northeastern states, along the foothills of the Himalayas, Jharkhand, West Orissa and Chhattisgarh, and on the eastern slopes of the Western Ghats. Teak is the most dominant species of this forest. Bamboos, sal, shisham, sandalwood, khair, kusum, arjun, mulberry are other commercially important species. The dry deciduous forests are found in areas having rainfall between 100 cm and 70 cm. These forests are found in the rainier parts of the peninsular plateau and the plains of Bihar and Uttar Pradesh. There are open stretches in which Teak, Sal, Peepal, Neem grow. A large part of this region has been cleared for cultivation and some parts are used for grazing.

Source) chapter 8, class 6th page no. 58-59 Feature)

Q.12) Ans) b Exp) Statement 1 is correct. The Inter Tropical Convergence Zone, or ITCZ, is a belt of low pressure which circles the Earth generally near the equator where the trade winds of the Northern and Southern Hemispheres come together. It is a broad trough of low pressure in equatorial latitudes. Statement 2 is correct. The Intertropical Convergence Zone (ITCZ), known by sailors as the doldrums or the calms because of its monotonous, windless weather, is the area where the northeast and southeast trade winds converge. It encircles Earth near the thermal equator, though its specific position varies seasonally. It shifts as the sun shifts between the Tropic of Cancer and Tropic of Capricorn. When it lies near the geographic Equator, it is called the near- equatorial trough. Where the ITCZ is drawn into and merges with a monsoonal circulation, it is sometimes referred to as a monsoon trough, a usage more common in Australia and parts of

ForumIAS Offline Guidance Centre 2nd Floor, IAPL House, 19, Pusa Road, Karol Bagh, New Delhi – 110005 | [email protected]|9821711605 https://t.me/UPSC_PDF www.upscpdf.com

PTS 2020|1|Test 4 - Solutions |ForumIAS

Asia including in the subcontinent. Therefore, Northern ITCZ is sometimes referred to as the Monsoonal trough. Statement 3 is correct. The location of the ITCZ gradually varies with seasons, roughly corresponding with the location of the sun (thermal equator). As the heat capacity of the oceans is greater than air over land, migration is more prominent over land. As land tends to heat up more the ITCZ can even shift beyond the Tropics. This can be seen during the heating up of the Tibetan plateau. As the Tibetan plateau heats up it leads to shifting of the Northern ITCZ beyond the Tropic of Cancer over the northern plains.

Source) NCERT- Class IX- SOCIAL SCIENCE CONTEMPORARY INDIA PART 1 CHAPTER 4, PAGE No. 31 Feature)

Q.13) Ans) c Exp) Option (c) is correct. Due to the apparent northward movement of the sun, the global heat belt shifts northward. As such, from March to May, it is hot weather season in India. Due to the hot weather, there is a flow of a strong, gusty, hot and dry wind over the northern plains. This is known as ‘Loo’. The low-pressure conditions also lead to the formation of dust storms and thunderstorms. They bring temporary relief as they lower the temperature and may bring light rain and a cool breeze. In West Bengal such thunderstorms are called as ‘Kaal Baisakhi’, calamity for the month of Baisakhi. Towards the close of the summer season, pre-monsoon showers are common especially in Kerala and Karnataka. They help in the early ripening of mangoes, and are often referred to as ‘mango showers’.

Source) NCERT- Class IX- SOCIAL SCIENCE CONTEMPORARY INDIA PART 1 CHAPTER 4, PAGE No. 33 Feature)

Q.14) Ans) c Exp) Option (c) is the correct answer. Amendments made to Section 215 (a) of the Tamil Nadu District Municipalities Act, 1920 and Building Rules 1973, have made it mandatory to provide rooftop rainwater harvesting structures in all new buildings. Rooftop Rainwater Harvesting is the technique through which

ForumIAS Offline Guidance Centre 2nd Floor, IAPL House, 19, Pusa Road, Karol Bagh, New Delhi – 110005 | [email protected]|9821711605 https://t.me/UPSC_PDF www.upscpdf.com

PTS 2020|1|Test 4 - Solutions |ForumIAS

rain water is captured from the roof catchments and stored in reservoirs. Harvested rain water can be stored in subsurface groundwater reservoir by adopting artificial recharge techniques to meet the household needs through storage in tanks.

The Main Objective of rooftop rainwater harvesting is to make water available for future use. Capturing and storing rainwater for use is particularly important in drylands, hilly, urban and coastal areas.

Source) Contemporary India-2, Chap-3, page-32

Q.15) Ans) c Exp) Option (c) is the correct answer. Statement 2 is correct. As per the RBI guidelines for Payment Banks, they cannot accept deposits of more than Rs 1 lakh from one account holder. Given that their primary role is to provide payments and remittance services and demand deposit products to small businesses and low-income households, payments banks are initially restricted to holding a maximum balance of Rs. 100,000 per individual customer. Statement 1 is incorrect. Payment banks are not allowed to lend money to their customers. They can accept demand deposits such as current deposits, and savings bank deposits from individuals, small businesses and other entities, as permitted. They do not accept time deposits. Statement 3 is incorrect. However, they may provide all other services such as ATM card, fund transfer, bill payments, recharges, net banking etc. to the account holders but they cannot issue credit cards. Payments and remittance services through various channels including branches, Automated Teller Machines (ATMs), Business Correspondents (BCs) and mobile banking can be done. KB) India Post Payments Bank (IPPB), which started functioning this month, is offering 3 types of zero-balance savings accounts with multiple benefits, including unlimited withdrawals and deposits besides doorstep banking. They are Digital, Basic and Regular.

Source) http://www.pib.nic.in/PressReleseDetail.aspx?PRID=1544722 https://rbi.org.in/scripts/bs_viewcontent.aspx?Id=2900

ForumIAS Offline Guidance Centre 2nd Floor, IAPL House, 19, Pusa Road, Karol Bagh, New Delhi – 110005 | [email protected]|9821711605 https://t.me/UPSC_PDF www.upscpdf.com

PTS 2020|1|Test 4 - Solutions |ForumIAS

Q.16) Ans) c Exp) Climate of any given region is determined by a variety of factors. These factors are also known as ‘controls of the climate’. There are six major controls of the climate of any place. They are: latitude, altitude, pressure and wind system, distance from the sea (continentality) and ocean currents and relief features. The amount of solar energy received varies according to the latitude. Higher the latitude more slanting are the sun’s rays thus an area at a higher latitude gets lower solar energy and is cooler than areas at a lower latitude. As a result, the temperature decreases from the equator towards the poles. Thus option (1) is correct. As one moves away from the surface of the earth to higher altitudes, the atmosphere becomes less dense which reduces the absorption and reflection capability of the atmosphere. The energy of the ‘Long wave Thermal Terrestrial Radiation’ coming from the earth’s surface thus dissipates as one moves to a higher altitude and leads to a decrease in temperature. The hills are therefore cooler during the summers as the atmosphere there is less dense. The pressure and wind system of any area depends on the latitude and altitude of the place. High pressure leads to dry and windless conditions, whereas low pressure leads to wet and windy conditions. Thus, it influences the temperature and rainfall pattern. Thus option (2) is correct. The sea exerts a moderating influence on climate. As the distance from the sea increases, its moderating influence decreases and the people experience extreme weather conditions. This condition is known as continentality (i.e. very hot during summers and very cold during winters). This happens because of the low Specific Heat Capacity of land as compared to that of water. This means that land heats up and cools down faster than water. Thus, during daytime land has a higher temperature and water in the oceans has a lower temperature, thus the wind blowing from the sea to the shore results in a moderating influence. Thus option (6) is correct. Ocean currents along with onshore winds affect the climate of the coastal areas, for example, any coastal area with warm or cold currents flowing past it, will be warmed or cooled due to the winds that are onshore. Similarly, relief of a region also plays a major role in determining the climate of that region. High mountains act as barriers to winds, thus moisture laden winds on striking these barriers results into rainfall. This is known as Orographic rainfall and occurs on the windward side of the mountains, whereas the dry air rises up and blows over the other side of the mountains which is known as the leeward side. Thus option (3) is the correct option. Longitude is used to calculate time at a place and has no effect on the climate. Though it can be used to pinpoint the exact location of the climate/weather, it has no role to play in the formation of the climate there.

ForumIAS Offline Guidance Centre 2nd Floor, IAPL House, 19, Pusa Road, Karol Bagh, New Delhi – 110005 | [email protected]|9821711605 https://t.me/UPSC_PDF www.upscpdf.com

PTS 2020|1|Test 4 - Solutions |ForumIAS

Vegetation is a result of climatic condition. Different types of climatic region have different vegetations. Thus it can be said that climatic conditions play an important role in the formation of vegetation of an area and not the other way around.

Source) NCERT- Class IX- SOCIAL SCIENCE CONTEMPORARY INDIA PART 1 CHAPTER 4, PAGE No. 27

Q.17) Ans) a Exp) In India, majority of the rainfall is received during the South West Monsoon (SWM) in the months of June, July, August and September. These SWM winds travel from the Indian Ocean to bring rainfall to the North East and the West coast of India. Statement 1 is incorrect. As we move away from north east towards the northern plains, the intensity of rainfall decreases. The SWM winds after striking the northeast turn towards the northern plains due to the ‘syntaxial bend of Himalayas’. Rainfall thus decreases from east to west in the northern plains because of progressive decrease in the humidity of the winds. As the moisture bearing winds of the Bay of Bengal branch moves further and further inland, they exhaust most of the moisture along the way. The regions of Assam, Meghalaya and Tripura receives higher rainfall as compared to the northern regions of UP, Bihar, Northern Madhya Pradesh etc. Statement no.2 is correct. The Coromandel coast of Andhra Pradesh and Tamil Nadu receives rainfall in the winter season due to northeast trade winds. A part of northeast trade wind blows over Bay of Bengal and gather moisture which causes rainfall in the coromandel coast. The normal annual rainfall of the state is about 945 mm (37.2 in) of which 48% is through the North East monsoon.

KB) During October-November, with the apparent movement of the sun towards the south, the monsoon trough or the low-pressure trough over the northern plains becomes weaker. This is gradually replaced by a high-pressure system. The south-west monsoon winds weaken and start withdrawing gradually. By the beginning of October, the monsoon withdraws from the Northern Plains. This leads to the onset of the North East winds from the high-pressure belt (now formed) over the northern plains and results in rainfall on the SouthEast coast of India (Coromandel coast - Andhra Pradesh and Tamil Nadu). In the second half of October, the mercury begins to fall rapidly in northern India. The low- pressure conditions, over northwestern India, get transferred to the Bay of Bengal by early November. This shift is

ForumIAS Offline Guidance Centre 2nd Floor, IAPL House, 19, Pusa Road, Karol Bagh, New Delhi – 110005 | [email protected]|9821711605 https://t.me/UPSC_PDF www.upscpdf.com

PTS 2020|1|Test 4 - Solutions |ForumIAS

associated with the occurrence of cyclonic depressions, which originate over the Andaman Sea. These cyclones generally cross the eastern coasts of India causing heavy and widespread rain. These tropical cyclones are often very destructive. The thickly populated deltas of the Godavari, the Krishna and the Kaveri are frequently struck by these cyclones, which causes great damage to life and property. The bulk of the rainfall of the Coromandel Coast is derived from depressions and cyclones.

Source) NCERT- Class IX- SOCIAL SCIENCE CONTEMPORARY INDIA PART 1 CHAPTER 4

Q.18) Ans) c Exp) Option (c) is the correct answer. The Friendship Bridge is an iconic project of the Maldives and China in co-building the 21st Century Maritime Silk Road and has been recently opened for traffic. This 2-km bridge connects Male (capital city of Maldives) and neighboring Hulhule island where the Maldives’ main international airport is located. The bridge has enabled tourists to transit between the two islands on land within five minutes. The China-Maldives Friendship Bridge can boost Malé’s economy, and position China as a leading investor in Maldives’ transport and trade for years to come.

Source) https://www.thehindu.com/news/international/china-hopes-maldives-will-back- belt-and-road-initiative-fta/article25038474.ece

Q.19) Ans) d Exp) The character and extent of vegetation are mainly determined by temperature along with humidity in the air, precipitation and soil. Temperature: On the slopes of the Himalayas and the hills of the Peninsula above the height of 915 metres, the fall in the temperature affects the types of vegetation and its growth, and changes it from tropical to subtropical to temperate and alpine vegetation. Photoperiod: The variation in duration of sunlight at different places is due to differences in latitude, altitude, season and the duration of the day. Due to longer duration of sunlight, trees grow faster in summer. Precipitation: In India almost, the entire rainfall is brought in by the advancing southwest monsoon (June to September) and retreating northeast monsoons. Areas of heavy rainfall have more dense vegetation as compared to other areas of less rainfall.

ForumIAS Offline Guidance Centre 2nd Floor, IAPL House, 19, Pusa Road, Karol Bagh, New Delhi – 110005 | [email protected]|9821711605 https://t.me/UPSC_PDF www.upscpdf.com

PTS 2020|1|Test 4 - Solutions |ForumIAS

Land (Relief): Land affects the natural vegetation directly and indirectly. The nature of land influences the type of vegetation. The fertile level is generally devoted to agriculture. The undulating and rough terrains are areas where grassland and woodlands develop and give shelter to a variety of wildlife. Soil: The soils also vary over space. Different types of soils provide basis for different types of vegetation. The sandy soils of the desert support cactus and thorny bushes while wet, marshy, deltaic soils support mangroves and deltaic vegetation. The hill slopes with some depth of soil have conical trees. Thus all the options are correct and hence answer is option (d).

Source) NCERT- Class IX- SOCIAL SCIENCE CONTEMPORARY INDIA PART 1 CHAPTER 5, PAGE No. 42,43

Q.20) Ans) b Exp) Option (b) is the correct answer.

Source) Map based

ForumIAS Offline Guidance Centre 2nd Floor, IAPL House, 19, Pusa Road, Karol Bagh, New Delhi – 110005 | [email protected]|9821711605 https://t.me/UPSC_PDF www.upscpdf.com

PTS 2020|1|Test 4 - Solutions |ForumIAS

Q.21) Ans) a Exp) Option (a) is the correct answer. ● The natural vegetation in an area depends on the temperature and moisture available. Different types of vegetation are dependent on difficult climatic conditions. ● Pair 3 is incorrectly matched. Tropical deciduous forests, also known as monsoon forests are less dense and shed their leaves at a particular time. They do this to conserve water during dry seasons. Important species found: sal, teak, peepal, neem and shisham ● Pair 1 is correctly matched. Thorny bushes, such type of vegetation is found in arid areas of the country. Leaves get modified into spines as adaptive measures to reduce the loss of water. Important species found: cactus, babool, keekar, khair ● Pair 2 is incorrectly matched. Mountain vegetation, is present at a height between 1500 and 2500 metres. Trees are mostly conical in shape. These trees are also called coniferous trees. Important species found: chir, pine, Deodar ● Pair 4 is incorrectly matched. Mangrove forests, trees in these forests survive in saline water. Found mainly in sundarbans in west bengal and in the Andamans and Nicobar Islands. Important species found are Sundari and Rhizophora.

Source) NCERT class 6 geography, chapter 8, page no. 57-61

Q.22) Ans) b Exp) Nokrek National Park: the core area of Nokrek Biosphere Reserve, is a national park located approximately 2 km from Tura Peak in West Garo Hills district of Meghalaya, India. UNESCO added this National park to its list of Biosphere Reserves in May 2009. Along with Balphakram national park, Nokrek is a hotspot of biodiversity in Meghalaya. Nokrek has a remnant population of the red panda that has generated curiosity across the world. Nokrek is also an important habitat of the Asian elephant. The park had eight species of cats, ranging from tiger to marbled cat but the current status of the former is uncertain. There is a vast range of plants found in the park. A huge canopy of thick, tall and lush green forests cover Nokrek and its environment. The mother germoplasm of Citrus indica (locally known as Memang Narang) have been discovered by science researchers within Nokrek Range. This discovery led to the establishment of the National Citrus Gene Sanctuary-cum- Biosphere Reserve covering an area of forty-seven square kilometres. The area has wild varieties of citrus fruits that provide a gene-pool for commercially produced citrus.

ForumIAS Offline Guidance Centre 2nd Floor, IAPL House, 19, Pusa Road, Karol Bagh, New Delhi – 110005 | [email protected]|9821711605 https://t.me/UPSC_PDF www.upscpdf.com

PTS 2020|1|Test 4 - Solutions |ForumIAS

Simlipal Biosphere reserve: Similipal National Park is a national park and tiger reserve in the Mayurbhanj district in the Indian state of Odisha. This reserve is part of the UNESCO World Network of Biosphere Reserves since 2009. Agasthyamalai Biosphere Reserve: It was established in 2001 and is present in Kerala and Tamil Nadu. Agasthyamalai Biosphere Reserve became part of World Network of Biosphere Reserves in 2016. Great Nicobar Biosphere Reserve: The Great Nicobar Biosphere Reserve encompasses a large part (some 85%) of the island of Great Nicobar, the largest of the Nicobar Islands. The Reserve has a total core area of approximately 885 km2, surrounded by a 12 km-wide "forest buffer zone".In year 2013 it was included in the list of Man and Biosphere program of UNESCO to promote sustainable development based on local community efforts and sound science

Source) NCERT- Class IX- SOCIAL SCIENCE CONTEMPORARY INDIA PART 1 CHAPTER 5, PAGE No. 50,51. Feature)

Q.23) Ans) c Exp) Option (c) is the correct answer. Statement 1 is correct. The Intergovernmental Oceanographic Commission of UNESCO (IOC- UNESCO) was established in 1960. It is a body with functional autonomy within UNESCO. It is the only competent organization for marine science within the UN system. Statement 2 is correct. India, along with 23 other Indian Ocean Nations, participated in a major Indian ocean-wide tsunami mock exercise (drill) on 4th& 5thSeptember, 2018. The Exercise, known as IOWave18, was organized by the Intergovernmental Oceanographic Commission (IOC) of UNESCO. IOC coordinated the setting up of the Indian Ocean Tsunami Warning and Mitigation System (IOTWMS) following the 26th December 2004 tsunami.

KB) The purpose of the Commission is to promote international cooperation and to coordinate programmes in research, services and capacity-building, in order to learn more about the nature and resources of the ocean and coastal areas and to apply that knowledge for the improvement of management, sustainable development, the protection of the marine environment, and the decision-making processes of its Member States. In addition, IOC is recognized through the United Nations Convention on the Law of the Sea (UNCLOS) as the competent international organization in the fields of Marine Scientific Research and Transfer of Marine Technology.

Source) http://pib.nic.in/newsite/PrintRelease.aspx?relid=183296

ForumIAS Offline Guidance Centre 2nd Floor, IAPL House, 19, Pusa Road, Karol Bagh, New Delhi – 110005 | [email protected]|9821711605 https://t.me/UPSC_PDF www.upscpdf.com

PTS 2020|1|Test 4 - Solutions |ForumIAS

http://www.ioc-unesco.org

Q.24) Ans) c Exp) Statement 1 is correct. Jet streams are a narrow belt of high altitude (above 12,000 m) winds in the troposphere. Their path has a meandering shape. The strongest jet streams are the polar jet streams. The Jet streams are a product of two factors : Solar insolation that forms atmospheric circulation of winds (Hadley cell etc) and the Coriolis force. Their speed varies from about 110 km/h in summer to about 184 km/h in winter. A number of separate jet streams have been identified. The most constant are the mid-latitude and subtropical jet stream. Statement is 2 correct. The western cyclonic disturbances are weather phenomena of the winter months brought in by the westerly flow (jet streams) from the Mediterranean region. They usually influence the weather of the north and north-western regions of India and bring about winter rainfall in these regions. They are large cyclonic systems that are very important with respect to harvesting of the rabi crops in the regions of Uttarakhand, Punjab, Haryana, northern Rajasthan and northwestern Uttar Pradesh. Statement 3 is incorrect. The ‘subtropical westerly jet stream’ is found over 27 - 30 north latitude. These jet streams blow south of Himalayas all throughout the year except in summer. In summer the subtropical westerly jet stream shifts north of Himalayas with the apparent movement of the sun towards the north of India. After the shift of the subtropical westerly jet stream northward, there is a sudden “burst” of monsoon in India. This leads to the formation of an easterly jet stream, called the ‘tropical easterly jet stream’ which blows over peninsular India, approximately over 14°N during the summer months.

Source) NCERT- Class IX- SOCIAL SCIENCE CONTEMPORARY INDIA PART 1 CHAPTER 4, PAGE No. 28

Q.25) Ans) a Exp) Option (a) is the correct answer. Statement 1 is correct. The International Commission on Irrigation and Drainage (ICID) was established in 1950. It is a leading scientific, technical, an international not-for-profit, non- governmental organization. ICID is a professional network of experts from across the world in the field of irrigation, drainage, and flood management. Statement 3 is incorrect. A meeting of the International Executive Council, the highest decision-making body of International Commission on Irrigation and Drainage (ICID), has accepted Telangana government’s nomination of Sadarmat anicut across river Godavari in

ForumIAS Offline Guidance Centre 2nd Floor, IAPL House, 19, Pusa Road, Karol Bagh, New Delhi – 110005 | [email protected]|9821711605 https://t.me/UPSC_PDF www.upscpdf.com

PTS 2020|1|Test 4 - Solutions |ForumIAS

Nirmal district and Pedda Cheruvu in Kamareddy district in the ICID Register of Heritage Irrigation Structures at Saskatoon in Canada. Thus statement 2 is correct.

KB) Its main mission is to promote ‘Sustainable agriculture water management’ to achieve ‘Water secure world free of poverty and hunger through sustainable rural development’. ICID initially had 11 countries as Founder Members (Brazil, Egypt, India, Indonesia, Italy, Netherlands, Serbia (former Yugoslavia), Sri Lanka, Switzerland, Thailand and Turkey). Its Secretariat (Central Office) is located in New Delhi.

Source) https://www.thehindu.com/news/national/telangana/heritage-tag-for-two-irrigation- facilities-in-telangana/article24910458.ece

Q.26) Ans) d Exp) Option (d) is the correct answer. The Western Ghats and the Eastern Ghats mark the western and the eastern edges of the Deccan Plateau respectively. Major differences between them are: Western Ghats: They lie parallel to the western coast. They are continuous and can be crossed through passes only. They are present in Kerala, Tamil Nadu, Karnataka, Goa, Maharashtra and Gujarat. The Western Ghats are higher than the Eastern Ghats. Their average elevation is 900– 1600 metres. The Western Ghats cause orographic rain as they act as a barrier to the moisture laden winds coming from the Arabian sea. The Western Ghats are known by different local names. Statement 1 correct. They are known as Sahyadri in northern Maharashtra, Sahya Parvatam in Kerala and Nilagiri Malai in Tamil Nadu. The height of the Western Ghats progressively increases from north to south. Statement 2 is correct. The highest peaks include the ‘AnaiMudi’ in Kerala (2,695metres) and the ‘DodaBetta’ in Tamil Nadu (2,637 metres). Eastern Ghats: The Eastern Ghats stretch from the Mahanadi Valley to the Nigiris in the south. The Eastern Ghats are discontinuous and irregular and dissected by rivers draining into the Bay of Bengal. The Eastern Ghats run from the northern Odisha through Andhra Pradesh to Tamil Nadu in the south passing some parts of Karnataka and in Wayanad district of Kerala. They are eroded and cut through by four major rivers of peninsular India, viz. Godavari, Mahanadi, Krishna, and Kaveri. Statement 2 is correct. Mahendragiri (1,501 metres) is the highest peak in the Eastern Ghats. Statement 3 is correct. Shevaroy Hills and the Javadi Hills are located to the southeast of the Eastern Ghats and form the rear end of the eastern ghats. Eastern and Western Ghats intersect at Nilgiri Hills. Source: NCERT Class IX- Social Science, Contemporary India – 1 (Geography); Chapter-2 (Physical Features of India), Page No. 13

ForumIAS Offline Guidance Centre 2nd Floor, IAPL House, 19, Pusa Road, Karol Bagh, New Delhi – 110005 | [email protected]|9821711605 https://t.me/UPSC_PDF www.upscpdf.com

PTS 2020|1|Test 4 - Solutions |ForumIAS

Q.27) Ans) d Exp) Indian cybercrime officials, tracking the recently detected Agent Smith malware, believe it is targeted at Indians, who constitute the highest number of victims so far. Agent Smith is embedded in apps available on Google Playstore, mostly connected to gaming, image editing or adult entertainment. Once a user downloads the app, the malware gets active, looking for other apps that it can take over. Its ability to impersonate apps, as well as the fact that its icon is not visible on the user’s screen, makes it next to impossible to detect. The structure of the malware, too, indicates that it is an advanced one. Unlike most malware, the creators of Agent Smith seem to have made the effort to identify all the latest vulnerabilities in the Android operating system and designed it specifically to exploit them.

Source) https://www.thehindu.com/news/cities/mumbai/cybercrime-officials-suspect-agent- smith-targets-indians/article28741194.ece

Q.28) Ans) a Exp) Option (a) is the correct answer. Refer to the map for these ranges.

ForumIAS Offline Guidance Centre 2nd Floor, IAPL House, 19, Pusa Road, Karol Bagh, New Delhi – 110005 | [email protected]|9821711605 https://t.me/UPSC_PDF www.upscpdf.com

PTS 2020|1|Test 4 - Solutions |ForumIAS

Himalayan Range is a mountain range separating the plains of the Indian subcontinent from the Tibetan Plateau. It runs west northwest to east southeast in an arc 2400 kilometers long. They are present around 28-35degN latitude. The Aravali Range is a mountain range running approximately 692 kilometers in a southwest direction, starting in North India from Delhi and passing through southern Haryana through to western India across the states of Rajasthan and ending in Gujarat. They are present around 25degN latitude. The Vindhyan Range is a complex, discontinuous chain of mountain ridges, hill ranges, highlands and plateau in the west central India. They are present around 24degN latitude. The Satpura range is a range of hills in central India. The range rises in eastern Gijarat state running through the border of Maharashtra and Madhya Pradesh to the east till chhattisgarh.The range parallels the vindhya Range to the north. They are present around 21degN latitude.

Source) NCERT- Indian Physical Environment of Class 11, chapter 7, page no. 50

Q.29) Ans) b Exp) Statement 1 is incorrect: In order to effectively address the heinous crimes of sexual abuse and sexual exploitation of children through less ambiguous and more stringent legal provisions, the Ministry of Women and Child Development championed the introduction of the Protection of Children from Sexual Offences (POCSO) Act, 2012. The Act defines a child as any person below eighteen years of age, and regards the best interests and well-being of the child as being of paramount importance at every stage, to ensure the healthy physical, emotional, intellectual and social development of the child. The National Commission for the Protection of Child Rights (NCPCR) was established in March 2007 as a statutory body under the Commissions for Protection of Child Rights Act, 2005. It was set up to protect, promote and defend child rights in the country. The Commission consists of a chairperson and six members who are well versed in child welfare. Statement 2 is correct. To prevent misuse of the law, punishment has been provided for making false complaint or providing false information with malicious intent. Such punishment has been kept relatively light (six months) to encourage reporting. If false complaint is made against a child, punishment is higher (one year) (Section 22).

KB) Some more features of POCSO Act: ● It defines different forms of sexual abuse, including penetrative and non-penetrative assault, as well as sexual harassment and pornography, and deems a sexual assault to

ForumIAS Offline Guidance Centre 2nd Floor, IAPL House, 19, Pusa Road, Karol Bagh, New Delhi – 110005 | [email protected]|9821711605 https://t.me/UPSC_PDF www.upscpdf.com

PTS 2020|1|Test 4 - Solutions |ForumIAS

be “aggravated” under certain circumstances, such as when the abused child is mentally ill or when the abuse is committed by a person in a position of trust or authority vis-à- vis the child, like a family member, police officer, teacher, or doctor. ● People who traffic children for sexual purposes are also punishable under the provisions relating to abetment in the Act. The Act prescribes stringent punishment graded as per the gravity of the offence, with a maximum term of rigorous imprisonment for life, and fine. ● The Act provides for the establishment of Special Courts for trial of offences under the Act, keeping the best interest of the child as of paramount importance at every stage of the judicial process. ● The Act incorporates child friendly procedures for reporting, recording of evidence, investigation and trial of offences. ● The Act recognizes that the Intent to commit an offence, even when unsuccessful for whatever reason, needs to be penalized. ● The attempt to commit an offence under the Act has been made liable for punishment for upto half the punishment prescribed for the commission of the offence. ● The Act also provides for punishment for abetment of the offence, which is the same as for the commission of the offence. ● The Act makes it mandatory to report commission of an offence and also the recording of complaint and failure to do so would make a person liable for punishment of imprisonment for six months or / and with fine. 18.1t is a Punishable action if Police / Special Juvenile Police Unit fails to report a commission of an offence under this act [Section- 2141)] ● For the more heinous offences of Penetrative Sexual Assault, Aggravated Penetrative Sexual Assault, Sexual Assault and Aggravated Sexual Assault, the burden of proof is shifted to the accused. This provision has been made keeping in view the greater vulnerability and innocence of children. ● For speedy trial, the Act provides for the evidence of the child to be recorded within a period of 30 days. Also, the Special Court is to complete the trial within a period of one year, as far as possible (Section 35) ● The Act casts a duty on the Central and State Governments to spread awareness through media including television, radio and the print media at regular intervals to make the general public, children as well as their parents and guardians aware of the provisions of this Act. ● The POCSO Act is only applicable to child survivors and adult offenders. In case two children have sexual relations with each other, or in case a child perpetrates a sexual

ForumIAS Offline Guidance Centre 2nd Floor, IAPL House, 19, Pusa Road, Karol Bagh, New Delhi – 110005 | [email protected]|9821711605 https://t.me/UPSC_PDF www.upscpdf.com

PTS 2020|1|Test 4 - Solutions |ForumIAS

offence on an adult, the Juvenile Justice (Care and Protection of Children) Act, 2000, will apply.

Source) https://www.thehindu.com/opinion/op-ed/cauldron-of-sexual- misconduct/article28775461.ece http://vikaspedia.in/education/policies-and-schemes/protection-of-children-from-sexual- offences-act

Q.30) Ans) b Exp) Drought is a complex phenomenon as it involves elements of meteorology like precipitation, evaporation, evapotranspiration, ground water, soil moisture, storage and surface run-off, agricultural practices, particularly the types of crops grown, socio-economic practices and ecological conditions. Types of Droughts: Meteorological Drought : It is a situation when there is a prolonged period of inadequate rainfall marked with mal-distribution of the same over time and space. Agricultural Drought : It is also known as soil moisture drought, characterised by low soil moisture that is necessary to support the crops, thereby resulting in crop failures. Moreover, if an area has more than 30 percent of its gross cropped area under irrigation, the area is excluded from the drought-prone category Hydrological Drought : It results when the availability of water in different storages and reservoirs like aquifers, lakes, reservoirs, etc. falls below what the precipitation can replenish. Ecological Drought : When the productivity of a natural ecosystem fails due to shortage of water and as a consequence of ecological distress, damages are induced in the ecosystem

Source) Class XI NCERT, Chapter 7, Page no. 70

Q.31) Ans) d Exp) Option (d) is the correct answer. The Himalayas, geologically young and structurally fold mountains stretch over the northern borders of India. These mountain ranges run in a west-east direction from the Indus to the Brahmaputra. Statement 1 and 2 are correct. The Himalaya consists of three parallel ranges in its longitudinal extent. A number of valleys lie between these ranges. The northernmost range is known as the Great or Inner Himalayas or the ‘Himadri’. It is the most continuous range

ForumIAS Offline Guidance Centre 2nd Floor, IAPL House, 19, Pusa Road, Karol Bagh, New Delhi – 110005 | [email protected]|9821711605 https://t.me/UPSC_PDF www.upscpdf.com

PTS 2020|1|Test 4 - Solutions |ForumIAS

consisting of the loftiest peaks with an average height of 6,000 metres. The range lying to the south of the Himadri forms the most rugged mountain system and is known as Himachal or lesser Himalaya. The ranges are mainly composed of highly compressed and altered rocks. The altitude varies between 3,700 and 4,500 metres with an average width of 50 Km. While the Pir Panjal range forms the longest and the most important range, the Dhaula Dhar and the Mahabharat ranges are also prominent ones. This range consists of the famous valley of Kashmir, Kangra and Kullu Valley in Himachal Pradesh. The outermost range of the Himalayas is called the Shiwaliks. They extend over a width of 10-50 Km and have an altitude varying between 900 and 1100 metres. The longitudinal valley lying between lesser Himalaya and the Shiwaliks are known as Duns. DehraDun, Kotli Dun and Patli Dun are some of the well-known Duns. Statement 3 is correct. The Brahmaputra marks the eastern most boundary of the Himalayas. Beyond the Dihang gorge, the Himalayas bend sharply to the south and spread along the eastern boundary of India. They are known as the Purvanchal or the Eastern hills and mountains. These hills running through the north-eastern states are mostly composed of strong sandstones which are sedimentary rocks. Covered with dense forests, they mostly run as parallel ranges and valleys. The Purvanchal comprises the Patkai hills, the Naga hills, Manipur hills and the Mizo hills.

KB) Himalayan Ranges Includes the Himalayas, Purvanchal and their extensions Arakan Yoma (Myanmar) and Andaman and Nicobar Islands (but we will consider these as islands only). It is the youngest and highly unstable landmass of India. [Continent – Continent Convergence] Tectonic movements are very common. Division of the Himalayas from south to north. Shiwaliks or outer Himalayas Lesser or Middle Himalayas The Greater Himalayas The Trans-Himalayas – Tibetan Himalayas. The Eastern Hills – Purvanchal: A chain of hills in North-East India Source) NCERT Class IX- Social Science, Contemporary India – 1 (Geography); Chapter-2 (Physical Features of India), Page No. 9-11

ForumIAS Offline Guidance Centre 2nd Floor, IAPL House, 19, Pusa Road, Karol Bagh, New Delhi – 110005 | [email protected]|9821711605 https://t.me/UPSC_PDF www.upscpdf.com

PTS 2020|1|Test 4 - Solutions |ForumIAS

Q.32) Ans) d Exp) Tea cultivation is an example of plantation agriculture. It is also an important beverage crop introduced in India initially by the British. Today, most of the tea plantations are owned by Indians. Statement 1 is incorrect. The tea plant grows well in tropical and sub-tropical climates endowed with deep and fertile well-drained soil, rich in humus and organic matter. Tea bushes require warm and moist frost-free climate all through the year. Statement 2 is incorrect. Frequent showers evenly distributed over the year ensure continuous growth of tender leaves. Tea is a labor-intensive industry. It requires abundant, cheap and skilled labour. Tea is processed within the tea garden to restore its freshness. Major tea producing states are Assam, hills of Darjeeling and Jalpaiguri districts, West Bengal, Tamil Nadu and Kerala. Apart from these, Himachal Pradesh, Uttarakhand, Meghalaya, Andhra Pradesh and Tripura are also tea- producing states in the country. Statement 3 is incorrect. India is the second largest producer of tea after China Source) NCERT Class X, Chapter 4, Page no. – 40 https://www.worldatlas.com/articles/the-worlds-top-10-tea-producing-nations.html

Q.33) Ans) b Exp) Statement 1 is incorrect: In December 1999, the Parliament had passed the Geographical Indications of Goods (Registration and Protection) Act,1999. This Act seeks to provide for the registration and better protection of geographical indications relating to goods in India. It confers legal protection to Geographical Indications in India and prevents unauthorised use of a Registered Geographical Indication by others.The Act would be administered by the ‘Controller General of Patents, Designs and TradeMarks’ - who is the Registrar of Geographical Indications. Statement 2 is correct: GIs have been defined under Article 22(1) of the WTO Agreement of TRIPS (Trade Related aspects of Intellectual Property Rights) Agreement as: "Indications which identify a good as originating in the territory of a member, or a region or a locality in that territory, where a given quality, reputation or characteristic of the good is essentially attributable to its geographic origin." Thus, GI tag is internationally recognised.

KB) Some recent GI tags are - Kandhamal Haldi (Odisha), Rasagola (Odisha), Sirsi Supari (Karnataka), Erode Turmeric (Tamil Nadu), Marayoor Jaggery (Tamil Nadu), Shahi Litchi (Bihar) etc.

ForumIAS Offline Guidance Centre 2nd Floor, IAPL House, 19, Pusa Road, Karol Bagh, New Delhi – 110005 | [email protected]|9821711605 https://t.me/UPSC_PDF www.upscpdf.com

PTS 2020|1|Test 4 - Solutions |ForumIAS

Source) https://indianexpress.com/article/lifestyle/food-wine/odisha-receives-gi-tag-for- rasagola-5860795/ http://vikaspedia.in/social-welfare/entrepreneurship/geographical-indications

Q.34) Ans) b Exp) The climate of India is strongly influenced by the South West Monsoon (SWM) winds. These flow from the south Indian Ocean towards the Indian subcontinent. They are responsible for the majority of rainfall in the subcontinent. They reach Indian shores during the month of June. This season is known as ‘Monsoon’ based on the arabic word ‘Mausim’. To understand the mechanism of monsoon in India, the following facts are important. (a) The differential heating and cooling of land and water creates low pressure on the landmass of India while the seas around experience comparatively high pressure. This leads to rise of warm air over the plains of North India which travels through the upper troposphere and after losing its energy it cools down and descends near 20°S in the southern Indian ocean. This descending and outflowing air results in the formation of SWM winds after it crosses the equator. (b) The shift of the position of Inter Tropical Convergence Zone (ITCZ) in summer, over the Ganga plain. ITCZ is an equatorial trough normally positioned about 5°N of the equator – also known as the monsoon trough during the monsoon season. It is a low-pressure zone that is formed over the Ganga plains as the Sun moves north to the Tropic of Cancer during the months of April, May and June. This low-pressure zone augments and strengthens the SWM winds originating in the southern Indian Ocean. Thus option (6) is incorrect. (c) The presence of the high-pressure area, east of Madagascar, approximately at 20°S over the Indian Ocean. The intensity and position of this high-pressure area affects the Indian Monsoon. This is caused due to the descending air from the upper troposphere. The greater is the Mascarene High the stronger are the SWM winds. (d) The Tibetan plateau gets intensely heated during summer, which results in strong vertical air currents. This leads to shifting of the ITCZ northwards and intensifies the low pressure above the plains. This in turn strengthens the Mascarene high and results in a stronger SWM winds.. (e) The movement of the westerly jet stream to the north of the Himalayas and the presence of the tropical easterly jet stream over the Indian peninsula during summer also results in the burst of the monsoon. Thus option (2) is incorrect.

Source) NCERT- Class IX- SOCIAL SCIENCE CONTEMPORARY INDIA PART 1 CHAPTER 4, PAGE No. 31

ForumIAS Offline Guidance Centre 2nd Floor, IAPL House, 19, Pusa Road, Karol Bagh, New Delhi – 110005 | [email protected]|9821711605 https://t.me/UPSC_PDF www.upscpdf.com

PTS 2020|1|Test 4 - Solutions |ForumIAS

Q.35) Ans) a Exp) Statement 1 is correct. Climate describes what the weather is like over a long period of time in a specific area. Different regions can have different climates. To describe the climate of a place, we might say what the temperatures are like during different seasons, how windy it usually is, or how much rain or snow typically falls. Weather refers to short-term changes in the atmosphere. Even though there’s only one atmosphere on Earth, the weather isn’t the same all around the world. Weather is different in different parts of the world and changes over minutes, hours, days, and weeks. Statement 2 is incorrect. The elements of weather and climate are the same, i.e. temperature, atmospheric pressure, wind, humidity and precipitation. At any point of time, the elements to decide the weather and climate of the region does not change, but remains constant.

Source) NCERT- Class IX- SOCIAL SCIENCE CONTEMPORARY INDIA PART 1 CHAPTER 4, PAGE No. 26

Q.36) Ans) c Exp) Option (c) is the correct answer. Statement 1 is correct. UNSDF 2018-2022 outlines development cooperation strategy between Union Government and United Nations Country Team in India in support of achievement of India’s key national development priorities and Sustainable Development Goals (SDGs).

Statement 2 is correct. The programmatic work outlined in the UNSDF targets the seven low- income states (Bihar, Chhattisgarh, Jharkhand, MP, Odisha, Rajasthan, UP), the North-East region, and the aspirational districts identified earlier this year by the NITI Aayog.

KB) NITI Aayog and United Nations in India signed the Sustainable Development Framework for 2018-2022 at a function in New Delhi.  It was framed following highly participative process, in consultation with government entities, civil society representatives, academia, and the private sector.  Focus areas include poverty and urbanization, health, water, and sanitation, education, climate change, nutrition and food security, clean energy, and disaster resilience; skilling, entrepreneurship, job creation, gender equality and youth development.

ForumIAS Offline Guidance Centre 2nd Floor, IAPL House, 19, Pusa Road, Karol Bagh, New Delhi – 110005 | [email protected]|9821711605 https://t.me/UPSC_PDF www.upscpdf.com

PTS 2020|1|Test 4 - Solutions |ForumIAS

 UNSDF also includes set of UN flagship programs that are aligned with major government schemes.  These flagship programs will be scalable innovative, multi-sectoral solutions to some of the most pressing development challenges that India faces and also serve as catalysts for increased investment of development finance.  UNSDF programmes range from affordable housing for poor to increasing access to clean energy in rural off-grid areas, protecting all children from vaccine-preventable diseases, providing quality education for all children and skilling for young people, especially young girls and ending stunting to improving child sex ratio.

Source)  http://pib.nic.in/newsite/PrintRelease.aspx?relid=183795  https://in.one.un.org/un-press-release/niti-aayog-and-united-nations-in-india-sign- sustainable-development-framework-for-2018-2022/

Q.37) Ans) d Exp) Statement 1 is incorrect. Rice: It is the staple food crop of a majority of the people in India. Our country is the second largest producer of rice in the world after China. It is a kharif crop which requires high temperature (above 25°C) and high humidity with annual rainfall above 100 cm. In the areas of less rainfall, it grows with the help of irrigation. Rice is grown in the plains of north and north-eastern India, coastal areas and the deltaic regions. Development of dense network of canal irrigation and tubewells have made it possible to grow rice in areas of less rainfall such as Punjab, Haryana and western Uttar Pradesh and parts of Rajasthan Statement 2 is incorrect. Wheat: This is the second most important cereal crop. It is the main food crop, in north and north-western part of the country. This rabi crop requires a cool growing season and bright sunshine at the time of ripening. It requires 50 to 75 cm of annual rainfall evenly distributed over the growing season. There are two important wheat-growing zones in the country – the Ganga-Satluj plains in the north-west and black soil region of the Deccan. The major wheat-producing states are Punjab, Haryana, Uttar Pradesh, Madhya Pradesh, Bihar and Rajasthan.

Source) NCERT Class X, Chapter 4, page no. 37.

ForumIAS Offline Guidance Centre 2nd Floor, IAPL House, 19, Pusa Road, Karol Bagh, New Delhi – 110005 | [email protected]|9821711605 https://t.me/UPSC_PDF www.upscpdf.com

PTS 2020|1|Test 4 - Solutions |ForumIAS

Q.38) Ans) b Exp) Innovating for Clean Air (IfCA), an initiative that aims to provide a unique measurement system for air quality by integrating satellite and sensor data, and extending support to India’s transition to electric vehicles, was launched in Bangalore. The two-year UK-India joint initiative is expected to provide stakeholders of the two countries opportunities to test interventions related to air quality and Electric Vehicle (EV) integration. The key partners of the programme are the Indian Institute of Science (IISc), Enzen, Project Lithium, Confederation of Indian Industries, C40 Cities and Clean Air Platform.

Source) https://indianexpress.com/article/cities/bangalore/bengaluru-uk-gets-unique-air- quality-measurement-system-ifca-electric-vehicles-5867410/

Q.39) Ans) c Exp) Human beings the world over have experienced disasters and have faced and lived with them. Now people are becoming aware and various steps have been initiated at different levels for mitigating the effects of disasters. Identification and classification of disasters is being considered as an effective and scientific step to deal promptly and efficiently with the disasters. Broadly, natural disasters can be classified under four categories. The table below summarizes classification of natural disaster.

Classification of Natural Disasters

Atmospheric Terrestrial Aquatic Biological

Blizzards Earthquakes Floods Plants and Animals as Thunderstorms Volcanic Tidal Waves colonisers (Locusts, Lightning Eruptions Ocean etc.). Tornadoes Landslides Currents Insects infestation— Tropical Cyclone Avalanches Storm Surge fungal, bacterial and Drought Subsidence Tsunami viral diseases such as Hailstorm Soil Erosion bird flu, dengue, etc. Frost, Heat Wave or Loo. Cold Waves, etc.

From the table we can find that drought is categorized as atmospheric natural disaster, so statement 2 is incorrect. Hence, Option C is the correct answer.

ForumIAS Offline Guidance Centre 2nd Floor, IAPL House, 19, Pusa Road, Karol Bagh, New Delhi – 110005 | [email protected]|9821711605 https://t.me/UPSC_PDF www.upscpdf.com

PTS 2020|1|Test 4 - Solutions |ForumIAS

Source) NCERT- Indian Physical Environment of Class 11, Chapter no 7, Page no. 79.

Q.40) Ans) a Exp) Himalayas extend in the east-west direction from the Indus gorge in the west to the Brahmaputra gorge in the east. Statement 2 is correct. Himalayan ranges take sharp southward bends at these gorges. These bends are called Syntaxial bends of the Himalayas. Statement 1 is incorrect. The western Syntaxial bend occurs near the Nanga Parbat where the Indus River has cut a deep gorge. Statement 3 is correct. The eastern Syntaxial bend occurs near the Namche Barwa. Statement 4 is correct. The Syntaxial bend forms a barrier to the oncoming moisture laden Southwest monsoon winds and thus forms an ‘orographic effect’ leading to the heavy rainfall in the Northeast region of the country.

Source) https://www.pmfias.com/himalayas-himalayan-ranges-shiwaliks-outer-himalayas- lesser-himalayas-greater-himalayas-trans-himalayas- purvanchal/#Syntaxial_Bends_of_the_Himalayas

Q.41) Ans) c Exp) Indian Council of Medical Research (ICMR) launched India’s first large-scale trial for two new tuberculosis (TB) vaccines named IMMUVAC and VPM1002. These vaccines aim to prevent infection (pre-exposure) or prevent primary progression or reactivation of latent TB infection (post-exposure), all of which have a critical role to play in India’s big fight against TB. Both vaccines have been manufactured by Indian pharmaceutical companies. Immuvac: It is also known as mycobacterium indicus pranii (MIP). It has been manufactured by Ahmedabad based Cadila Pharmaceuticals. VPM1002: It is manufactured by Pune based Serum Institute of India.

Source) https://indianexpress.com/article/explained/explained-whats-at-stake-in-indias- biggest-ever-trial-of-tuberculosis-vaccines-5867729/

Q.42) Ans) d Exp) Option (d) is the correct option. Both the statements are incorrect. Seif is a long, narrow sand dune or chain of dunes, generally oriented in a direction parallel to the prevailing wind or in a direction resulting from two or more winds blowing at acute angles to

ForumIAS Offline Guidance Centre 2nd Floor, IAPL House, 19, Pusa Road, Karol Bagh, New Delhi – 110005 | [email protected]|9821711605 https://t.me/UPSC_PDF www.upscpdf.com

PTS 2020|1|Test 4 - Solutions |ForumIAS

each other. The dune crest consists of a series of peaks and gaps, and the steep, or slip, face may change sides of the dune according to changes in wind direction. Most Seif dunes occur in the open desert.

Barchan, also spelled Barkhan, crescent-shaped sand dune produced by the action of wind predominately from one direction. One of the commonest types of dunes, it occurs in sandy deserts all over the world. Barchans are convex facing the wind, with the horns of the crescent pointing downwind and marking the lateral advancement of the sand.

Source) Class XI NCERT, Chapter 2, Page no. - 17

Q.43) Ans) d Exp) Option (d) is the correct answer. Statement 1 is correct. Ayushman Bharat - National Health Protection Mission has included the on-going centrally sponsored schemes - Rashtriya Swasthya Bima Yojana and the Senior Citizen Health Insurance Scheme under its ambit.

ForumIAS Offline Guidance Centre 2nd Floor, IAPL House, 19, Pusa Road, Karol Bagh, New Delhi – 110005 | [email protected]|9821711605 https://t.me/UPSC_PDF www.upscpdf.com

PTS 2020|1|Test 4 - Solutions |ForumIAS

Statement 2 is correct. Ayushman Bharat is a National Health Protection Scheme which covers over 10 crore poor and vulnerable families (approximately 50 crore beneficiaries) providing coverage upto 5 lakh rupees per family per year for secondary and tertiary care hospitalization. Statement 3 is correct. Benefits of the scheme are portable across the country and a beneficiary covered under the scheme is allowed to take cashless benefits from any public/private enlisted hospitals across the country.

KB) Salient features of Ayushman Bharat include:  Ayushman Bharat - National Health Protection Mission will be an entitlement-based scheme with entitlement decided on the basis of deprivation criteria in the SECC database.  The beneficiaries can avail benefits in both public and empanelled private facilities.  To control costs, the payments for treatment will be done on package rate (to be defined by the Government in advance) basis.  One of the core principles of Ayushman Bharat - National Health Protection Mission is to cooperative federalism and flexibility to states.  For giving policy directions and fostering coordination between the Centre and States, it is proposed to set up Ayushman Bharat National Health Protection Mission Council (AB- NHPMC) at apex level Chaired by Union Health and Family Welfare Minister.  States would need to have State Health Agency (SHA) to implement the scheme.  To ensure that the funds reach SHA on time, the transfer of funds from Central Government through Ayushman Bharat - National Health Protection Mission to State Health Agencies may be done through an escrow account directly.  In partnership with NITI Ayog, a robust, modular, scalable and interoperable IT platform will be made operational which will entail a paperless, cashless transaction.

Source) http://pib.nic.in/newsite/PrintRelease.aspx?relid=183635

Q.44) Ans) b Exp) Statement 1 is correct. The months of October and November are known for retreating monsoons. By the end of September, the southwest monsoon becomes weak as the low-pressure trough of the Ganga plain starts moving southward in response to the southward movement of the sun.

ForumIAS Offline Guidance Centre 2nd Floor, IAPL House, 19, Pusa Road, Karol Bagh, New Delhi – 110005 | [email protected]|9821711605 https://t.me/UPSC_PDF www.upscpdf.com

PTS 2020|1|Test 4 - Solutions |ForumIAS

By the beginning of October, the low pressure covers northern parts of the Bay of Bengal and by early November, it moves over Karnataka and Tamil Nadu. By the middle of December, the centre of low pressure is completely removed from the Peninsula. The retreating Southwest monsoon season is marked by clear skies and rise in temperature. The land is still moist. Owing to the conditions of high temperature and humidity, the weather becomes rather oppressive. This is commonly known as the ‘October heat’. In the second half of October, the mercury begins to fall rapidly, particularly in northern India. The weather in the retreating monsoon is dry in north India but it is associated with rain in the eastern part of the Peninsula. Here, October and November are the rainiest months of the year. Hence statement 2 is incorrect. Statement 3 is correct because during the winter monsoon or retreating monsoon, the ITCZ moves southward, and so the reversal of winds from Northeast to South and southwest, takes place. So, option B is correct.

Source) NCERT- Indian Physical Environment of Class 11, Chapter no 4, Page no. 49. Feature)

Q.45) Ans) a Exp) Option (a) is the correct answer. Statement 1 and 2 are incorrect. Government e Marketplace is the National Public Procurement Portal offering an online, end-to-end solution for procurement of common use goods and services by government departments. GeM provides an open, inclusive, transparent and efficient online marketplace. Statement 3 is incorrect. It is used by Central and State Government Ministries and Departments, Central and State Public Sector Undertakings (CPSUs & SPSUs), autonomous institutions and local bodies.

KB) The National Mission on GeM (NMG) was launched on 5th September 2018 till 17th October 2018. The objective of the Mission was to accelerate the adoption and use of Procurement by Major Central Ministries, States and UTs and their agencies (including CPSUs/PSUs, Local Bodies) on the GeM platform. The objectives GeM are: Promote inclusiveness by catapulting various categories of sellers and service providers Highlight and communicate the ‘value addition’ by way of transparency and efficiency in public procurement, including corruption free governance.

ForumIAS Offline Guidance Centre 2nd Floor, IAPL House, 19, Pusa Road, Karol Bagh, New Delhi – 110005 | [email protected]|9821711605 https://t.me/UPSC_PDF www.upscpdf.com

PTS 2020|1|Test 4 - Solutions |ForumIAS

Achieve cashless, contactless and paperless transaction in line with Digital India objectives. Increase overall efficiency leading to significant cost savings on government expenditure in Procurement. Maximizing ease in availability of all types of products and services bought by the Government.

Source) http://pib.nic.in/PressReleseDetail.aspx?PRID=1547186

Q.46) Ans) c Exp)

Source) India: Physical Environment (NCERT Geography Class XI) CHAPTER-1

Q.47) Ans) d Exp) Option (d) is the correct answer. Statement 1 and 3 are correct. e-Vidhan is a Mission Mode Project to digitize and make the functioning of State Legislatures paperless. It is a part of Digital India programme. Ministry of

ForumIAS Offline Guidance Centre 2nd Floor, IAPL House, 19, Pusa Road, Karol Bagh, New Delhi – 110005 | [email protected]|9821711605 https://t.me/UPSC_PDF www.upscpdf.com

PTS 2020|1|Test 4 - Solutions |ForumIAS

Parliamentary Affairs, being the Nodal Ministry for this project, will roll out e-Vidhan as NeVA covering all 40 Houses including two Houses of Parliament and thereby putting them all on a single platform. This application is expected to prove the theory of ‘One Nation One Application’. Statement 2 is correct. It is a work-flow based app deployed on the Cloud (Meghraj) which helps the Chair of the House to conduct the proceedings of the House smoothly and the members to carry out their duties in the House efficiently.

KB) In order to utilise and harness the benefits of Cloud Computing, Government of India has embarked upon an ambitious initiative - "GI Cloud" which has been named as "MeghRaj". The focus of this initiative is to accelerate the delivery of e-services in the country while optimizing ICT spending of the Government. This will ensure optimum utilization of the infrastructure and speed up the development and deployment of eGov applications.

Source) http://pib.nic.in/newsite/PrintRelease.aspx?relid=183646

Q.48) Ans) b Exp) According to state records, the forest area covers 23.28 per cent of the total land area of the country. It is important to note that the forest area and the actual forest cover are not the same.

Forest Cover Forest Area

Forest cover is the area The area notified and recorded as occupied by forests with the forest land irrespective of the canopy. existence of trees

Forest cover is based on It is based on the records of the aerial photographs and State Revenue Department satellite imageries

Hence statement 1 and 2 are correct and statement 3 is incorrect. So, option B is the correct answer.

KB) According to India’s State of Forest Report 2011, the actual forest cover in India is only 21.05 per cent. Of the forest cover, the share of dense and open forests is 12.29 and 8.75 per cent respectively.

ForumIAS Offline Guidance Centre 2nd Floor, IAPL House, 19, Pusa Road, Karol Bagh, New Delhi – 110005 | [email protected]|9821711605 https://t.me/UPSC_PDF www.upscpdf.com

PTS 2020|1|Test 4 - Solutions |ForumIAS

Both forest area and forest cover varies from state to state. Lakshadweep has zero percent forest area; Andaman and Nicobar Islands have 86.93 per cent. Most of the states with less than 10 percent of the forest area lie in the north and northwestern part of the country. These are Rajasthan, Gujarat, Punjab, Haryana and Delhi. Most of the forests in Punjab and Haryana have been cleared for cultivation. States with 10-20 per cent forest area are Tamil Nadu and West Bengal. In Peninsular India, excluding Tamil Nadu, Dadra and Nagar Haveli and Goa, the area under forest cover is 20-30 per cent. The northeastern states have more than 30 percent of the land under forest. Hilly topography and heavy rainfall are good for forest growth.

Source) NCERT- Indian Physical Environment of Class 11, Chapter no 5, Page no. 61.

Q.49) Ans) b Exp) Option (b) is the correct answer. Option 1 is correct. Wular Lake - Wular Lake (also spelt Wullar) is one of the largest fresh water lakes in Asia. It is sited in Bandipora district in the Indian union territory of Jammu and Kashmir. The lake basin was formed as a result of tectonic activity and is fed by the Jhelum River. In addition, much of the lake has been drained as a result of willow plantations being built on the shore in the 1950s. Option 2 is correct. Pangong Tso Lake - Pangong Tso Tibetan for "high grassland lake", also referred to as Pangong Lake, is an endorheic lake in the Himalayas situated at a height of about 4,350 m (14,270 ft). It is 134 km (83 mi) long and extends from India to the Tibetan Autonomous Region, China. Approximately 60% of the length of the lake lies within the Tibetan Autonomous Region. During winter the lake freezes completely, despite being saline water. It is not a part of the Indus river basin area and geographically a separate landlocked river basin. The lake is in the process of being identified under the Ramsar Convention as a wetland of international importance. This will be the first trans-boundary wetland in South Asia under the convention. Option 3 is correct. Loktak Lake - It is the largest freshwater lake in Northeast India and is famous for the phumdis (heterogeneous mass of vegetation, soil and organic matter at various stages of decomposition) floating over it. The lake is located at Moirang in Manipur state, India. Located on this phumdi, Keibul Lamjao National Park is the only floating national park in the world. The park is the last natural refuge of the endangered Sangai (state animal). This ancient lake plays an important role in the economy of Manipur. It serves as a source of water for hydropower generation, irrigation and drinking water supply. The lake is also a source of livelihood for the rural fishermen who live in the surrounding areas and on phumdis, also known as "phumshongs". The lake was initially designated as a wetland of international importance under the Ramsar Convention on 23 March 1990. It was also listed under the Montreux Record

ForumIAS Offline Guidance Centre 2nd Floor, IAPL House, 19, Pusa Road, Karol Bagh, New Delhi – 110005 | [email protected]|9821711605 https://t.me/UPSC_PDF www.upscpdf.com

PTS 2020|1|Test 4 - Solutions |ForumIAS

on 16 June 1993, "a record of Ramsar sites where changes in ecological character have occurred, are occurring or are likely to occur". Option 4 is incorrect. Kolleru Lake – It is one of the largest freshwater lakes in India located in state of Andhra Pradesh and forms the largest shallow freshwater. Kolleru is located between Krishna and Godavari deltas. The lake is fed directly by water from the seasonal Budameru and Tammileru streams, and is connected to the Krishna and Godavari irrigation systems. Many birds migrate here in winter, such as Siberian crane, ibis, and painted storks. The lake was an important habitat for an estimated 20 million resident and migratory birds, including the grey or spot-billed pelican (Pelecanus philippensis). The lake was declared as a wildlife sanctuary in November 1999 under India's Wildlife Protection Act of 1972, and designated a wetland of international importance in November 2002 under the international Ramsar Convention.

Source) Class XI NCERT and Wikipedia

Q.50) Ans) c Exp) Option (c) is the correct answer. Statement 1 is correct. The key objectives of the policy are: ● Broadband for all; ● Creating four million additional jobs in the Digital Communications sector; ● Enhancing the contribution of the Digital Communications sector to 8% of India's GDP from ~ 6% in 2017; ● Propelling India to the Top 50 Nations in the ICT Development Index of ITU from 134 in 2017; ● Enhancing India's contribution to Global Value Chains; and ● Ensuring Digital Sovereignty. Statement 2 is correct. The policy advocates: ● Establishment of a National Digital Grid by creating a National Fibre Authority; ● Establishing Common Service Ducts and utility corridors in all new city and highway road projects; ● Creating a collaborative institutional mechanism between Centre, States and Local Bodies for Common Rights of Way, standardization of costs and timelines; ● Removal of barriers to approvals; and ● Facilitating development of Open Access Next Generation Networks.

Features: The policy aims to

ForumIAS Offline Guidance Centre 2nd Floor, IAPL House, 19, Pusa Road, Karol Bagh, New Delhi – 110005 | [email protected]|9821711605 https://t.me/UPSC_PDF www.upscpdf.com

PTS 2020|1|Test 4 - Solutions |ForumIAS

● Provide universal broadband connectivity at 50 Mbps to every citizen; ● Provide 1 Gbps connectivity to all Gram Panchayats by 2020 and 10 Gbps by 2022; ● Ensure connectivity to all uncovered areas; ● Attract investments of USD 100 billion in the Digital Communications Sector; ● Train one million manpower for building New Age Skills; ● Expand IoT ecosystem to 5 billion connected devices; ● Establish a comprehensive data protection regime for digital communications that safeguards the privacy, autonomy and choice of individuals ● Facilitate India's effective participation in the global digital economy; ● Enforce accountability through appropriate institutional mechanisms to assure citizens of safe and ● Secure digital communications infrastructure and services.

Source) http://pib.nic.in/newsite/PrintRelease.aspx?relid=183711

Q.51) Ans) c Exp) Recently workers were trapped in a Rat hole mine in Meghalaya for 18 days. Statement 1 is correct. Most of the minerals in India are nationalised and their extraction is possible only after obtaining due permission from the government. But in most of the tribal areas of the north-east India, minerals are owned by individuals or communities. Statement 2 is correct. In Meghalaya, there are large deposits of coal, iron ore, limestone and dolomite etc. Coal mining in Jowai and Cherapunjee is done by family members in the form of a long narrow tunnel, known as ‘Rat hole’ mining. It is a primitive and hazardous method of mining for coal, with tunnels that are only 3-4 feet in diameter (hence, rat-hole), leading to pits ranging from 5-100 sq. mt deep. There are two types of rat-holes: when dug into the ground these are vertical shafts leading to the mines where horizontal tunnels are dug; the second type is where horizontal holes are dug directly in the hillsides to reach coal seams (bed of coal). The coal is taken out manually, loaded into a bucket or a wheelbarrow and dumped on a nearby un-mined area. From here, it is carried to larger coal dumps near highways for trade and transportation The National Green Tribunal has declared such activities illegal and recommended that these should be stopped forthwith. The National Green Tribunal banned rat-hole mining in Meghalaya in 2014 on a petition that said acidic discharge from the mines was polluting the Kopili river. But the practice continues unabated.

Source) Chap-5, page no.-50, Contemporary India-2

ForumIAS Offline Guidance Centre 2nd Floor, IAPL House, 19, Pusa Road, Karol Bagh, New Delhi – 110005 | [email protected]|9821711605 https://t.me/UPSC_PDF www.upscpdf.com

PTS 2020|1|Test 4 - Solutions |ForumIAS

https://www.livemint.com/Industry/zoP2McbcYODv5J4CnhrveL/The-high-human-ecological- cost-of-rat-hole-mining.html

Q.52) Ans) d Exp) Option D is the correct answer. Black soil covers most of the Deccan Plateau which includes parts of Maharashtra, Madhya Pradesh, Gujarat, Andhra Pradesh and some parts of Tamil Nadu. In the upper reaches of the Godavari and Krishna, and the north western part of the Deccan Plateau, the black soil is very deep. These soils are also known as the ‘Regur Soil’ or the ‘Black Cotton Soil’. The black soils are generally clayey, deep and impermeable. They swell and become sticky when wet and shrink when dried. So, during the dry season, these soils develop wide cracks. Thus, there occurs a kind of ‘self-ploughing’. Because of this character of slow absorption and loss of moisture, the black soil retains the moisture for a very long time, which helps the crops, especially the rain fed ones, to sustain even during the dry season. Black soil is rich in lime, iron, magnesia and alumina. At the same time, it also contains potash. However black soil lacks phosphorus, nitrogen and organic matter. It is suitable for growing cotton.

Source) NCERT- Indian Physical Environment of Class 11, Chapter no 6, Page no. 69 and 71. Feature)

Q.53) Ans) c Exp) Earthquakes are by far the most unpredictable and highly destructive of all the natural disasters. Earthquakes that are of tectonic origin have proved to be the most devastating and their area of influence is also quite large. These earthquakes result from a series of earth movements brought about by a sudden release of energy during the tectonic activities in the earth’s crust. As compared to these, the earthquakes associated with volcanic eruptions, rock fall, landslides, subsidence, particularly in the mining areas, impounding of dams and reservoirs, etc. have limited area of influence and the scale of damage. Statement 1 is correct. National Geophysical Laboratory, Geological Survey of India, Department of Meteorology, Government of India, along with the recently formed National Institute of Disaster Management, have made an intensive analysis of more than 1,200 earthquakes that have occurred in India in different years in the past, and based on these, they divided India into the following five earthquake zones: I. Very high damage risk zone II. High damage risk zone

ForumIAS Offline Guidance Centre 2nd Floor, IAPL House, 19, Pusa Road, Karol Bagh, New Delhi – 110005 | [email protected]|9821711605 https://t.me/UPSC_PDF www.upscpdf.com

PTS 2020|1|Test 4 - Solutions |ForumIAS

III. Moderate damage risk zone IV. Low damage risk zone V. Very low damage risk zone. Out of these, the first two zones had experienced some of the most devastating earthquakes in India. Uttarakhand, Western Himachal Pradesh and Kashmir Valley in the Himalayan region and the Kuchchh (Gujarat) are included in the Very High Damage Risk Zone. The remaining parts of Jammu and Kashmir, Himachal Pradesh, Northern parts of Punjab, Eastern parts of Haryana, Delhi, Western Uttar Pradesh, and Northern Bihar fall under the High Damage Risk Zone. Remaining parts of the country fall under moderate to very Low Damage Risk Zone. So, statement 2 is correct. Hence, option (c) is the correct answer.

Source) NCERT- Indian Physical Environment of Class 11, Chapter no 7, Page no. 81 Feature)

Q.54) Ans) d Exp) Option (d) is the correct answer.

ForumIAS Offline Guidance Centre 2nd Floor, IAPL House, 19, Pusa Road, Karol Bagh, New Delhi – 110005 | [email protected]|9821711605 https://t.me/UPSC_PDF www.upscpdf.com

PTS 2020|1|Test 4 - Solutions |ForumIAS

Statement 1 is correct. Over the 5 years project duration, e-VBAB Network Project will provide free tele-education courses in various academic disciplines to 4000 students every year from African countries. Statement 2 is correct. The Project will also be utilized for providing free continuing Medical Education (1000 every year) to African doctors/nurses/para-medical staff. Further, Indian doctors, through this project will provide free medical consultancy to those African doctors who seek such consultancy. In order to operationalize the e-VBAB Network Project, a Data Centre and Disaster Recovery Centre will be established in India along with Learning Centres in various African countries which decide to be part of the Project. There will be two separate platforms for e-Vidya Bharati (tele-education) and e-Aarogya Bharati (tele-medicine) which will link, through a web-based technology, various educational institutions and hospitals in India and the participating African countries. Statement 3 is correct. The e-VBAB Network Project will be completely funded by the Government of India for its entire duration and will be open for participation to all our partner countries in Africa. The Project will be another important milestone in our development partnership with Africa.

KB) Ministry of External Affairs (MEA) and the Telecommunications Consultants India Ltd (TCIL) signed an Agreement for the implementation of e-VidyaBharati and e-AarogyaBharati (e-VBAB) Network Project in 2018. E-VBAB Network Project is primarily a technological upgrade and extension of the Pan-African e-Network Project (Phase 1) which was implemented in 48 partner countries across Africa from 2009 till 2017. The Phase 1 of the Project successfully imparted tele-education and tele-medicine by linking educational institutions and hospitals in India with those from the participating African countries.

Source) https://www.business-standard.com/article/news-ani/tcil-mea-sign-mou-for-pan- africa-e-network-project-118091200368_1.html

Q.55) Ans) b Exp) The prominent mountain passes of India are: 1) Nathu La Pass: Located in Sikkim it is a border trade post between India and China. 2) Shipki La Pass:It is in Himachal Pradesh and is also a trade post between India and China. It is located through Sutlej Gorge. 3) Jelep La Pass: This pass links Sikkim to Lhasa in Tibet autonomous region of China. It passes through the Chumbi valley which was in the news due to Dokhlam stand-off.

ForumIAS Offline Guidance Centre 2nd Floor, IAPL House, 19, Pusa Road, Karol Bagh, New Delhi – 110005 | [email protected]|9821711605 https://t.me/UPSC_PDF www.upscpdf.com

PTS 2020|1|Test 4 - Solutions |ForumIAS

4) Khardung La:It links Leh to Siachen. It is the highest motorable pass in the country.. 5) Bara-Lacha La: This pass links Himachal Pradesh to Ladakh, connecting Manali with Leh. 6) Lipu Lekh:This pass connects Uttarakhand with Tibet autonomous region of China. Mansarovar pilgrims travel through this pass. It is also a border trade post between India and China. 7) Niti Pass:This pass joins Tibet ARC with Uttarakhand. 8) Rohtang Pass:It is located on eastern Pir Panjal Range and connects Kullu valley with Lahaul and Spiti valley of Himachal Pradesh. 9) Bomdi-La:The Bomdi-La pass connects Arunachal Pradesh with Lhasa, in Tibet ARC. 10) Dihang pass: Located in Arunachal Pradesh, this pass provides connectivity with Myanmar. 11) Diphu pass:This pass also connects Arunachal Pradesh with Myanmar. It remains open all round the year and is hence important for trade and transportation. 12) Zoji La:It connects Srinagar with Kargil and Leh. Beacon Force of Border Road Organization is responsible for clearing and maintaining this pass. 13) Senkotta Gap:It is located in the Western Ghats and connects Madurai in Tamil Nadu with Kottayam in Kerala.

Source) India: Physical Environment (NCERT Geography Class XI) CHAPTER-1

ForumIAS Offline Guidance Centre 2nd Floor, IAPL House, 19, Pusa Road, Karol Bagh, New Delhi – 110005 | [email protected]|9821711605 https://t.me/UPSC_PDF www.upscpdf.com

PTS 2020|1|Test 4 - Solutions |ForumIAS

Q.56) Ans) b Exp) Soil is one of the most important natural resources that nature has provided to humankind. It actually sustains life on the planet as it is an essential ingredient for growing plants. Laterite Soil Such soils are found in regions of high temperature and very high rainfall. The rains will wash all the nutrients from the soil, leaving it infertile. The leaching process will leave behind iron-rich soil, devoidt of silicon. These soils are poor in organic matter, nitrogen, phosphate and calcium. So, statement 1 is incorrect. Red Soil and Yellow Soil Red soil gets its colour from the iron found in its composition in a crystallized form. The soil takes on a yellow colour when it is hydrated. These soils are generally found in the Western Ghats, Odisha and Chattisgarh. Red and yellow soil is generally poor in nitrogen, phosphorous and humus. So, statement 2 is correct. Peaty Soil These are soils that have a very high content of organic matter and humus. They are seen in regions with high rainfall and average temperatures. If there is a lot of vegetation around the dead organic matter will accumulate, and in turn, enrich the soil with humus. These soils also have a black colour and are loamy in texture. There are usually found in parts of Bihar, Bengal and Tamil Nadu. And while they are suitable for agriculture, these soils need proper care so they do not leach away all their nutrients in the heavy rainfall. So, statement 3 is correct. Hence, option B is the correct answer.

Source) NCERT- Indian Physical Environment of Class 11, Chapter no 6, Page no. 71 and 72

Q.57) Ans) d Exp) Option (d) is the correct answer. Statement 1 is correct. Ministry of Power, Government of India has launched an ambitious program to encourage the deployment of Energy Efficient chiller systems in the country. The Chiller Star Labelling Program has been formulated by the Bureau of Energy Efficiency (BEE). The program envisages providing star rating in terms of its energy performance. Statement 2 is correct. Chillers are used extensively for space conditioning of buildings and for industrial process cooling applications. Statement 3 is correct. Chillers, being energy intensive system, contribute more than 40 per cent of the total energy consumption in commercial buildings. Therefore, it is important to

ForumIAS Offline Guidance Centre 2nd Floor, IAPL House, 19, Pusa Road, Karol Bagh, New Delhi – 110005 | [email protected]|9821711605 https://t.me/UPSC_PDF www.upscpdf.com

PTS 2020|1|Test 4 - Solutions |ForumIAS

optimize energy performance of chillers and create awareness amongst the end users to adopt transition towards energy efficient chillers.

Source) http://pib.nic.in/newsite/PrintRelease.aspx?relid=183458

Q.58) Ans) a Exp) India’s land frontier touches 17 states. The largest boundary that India has is with Bangladesh, followed by with China, Pakistan, Nepal, Myanmar, and Bhutan. Hence, statement 1 is correct. India has a long mainland coastline of 6100km and 7517km including the island groups of Andaman and Nicobar and Lakshadweep. The coastal states are - Gujarat, Maharashtra, Goa, Kerala, Tamil Nadu, Andhra Pradesh,Karnataka, Odisha, West Bengal. Gujarat has the longest coastline. Hence, statement 2 is correct. The Tropic of Cancer passes through Gujarat, Rajasthan, Madhya Pradesh, Chhattisgarh, Jharkhand, West Bengal, Tripura, and Mizoram. Statement 3 is incorrect.

Source) India: Physical Environment (NCERT Geography Class XI) CHAPTER-1

ForumIAS Offline Guidance Centre 2nd Floor, IAPL House, 19, Pusa Road, Karol Bagh, New Delhi – 110005 | [email protected]|9821711605 https://t.me/UPSC_PDF www.upscpdf.com

PTS 2020|1|Test 4 - Solutions |ForumIAS

Q.59) Ans) c Exp) India follows a single standard time knows as ‘Indian Standard Time’ which is GMT+30. The 82.5’E longitude which passes through Mirzapur near Allahabad is used for its measurement. India does not observe daylight savings time. They were briefly used during the Sino–Indian War of 1962 and the Indo–Pak Wars of 1965 and 1971. Hence, statement 1 is correct. CSIR-NPL monitors the Indian Standard Time along with maintaining other standards. It’s suggestion for a different time zone for the north eastern states was recently denied by the government. Statement 2 is correct. Bagaan time or Chai Bagaan time is used by tea gardens in Assam to save their daylight working hours. It is one hour ahead of the IST. Statement 3 is incorrect.

KB) Daylight saving time (DST), also daylight savings time or daylight time (United States) and summer time (United Kingdom, European Union, and others), is the practice of advancing clocks during the summer months so that evening daylight lasts longer, while sacrificing normal sunrise times. Typically, regions that use daylight saving time adjust clocks forward one hour close to the start of spring and adjust them backward in the autumn. In effect, DST causes a lost hour of sleep in the spring and an extra hour of sleep in the fall.

Source) India: Physical Environment (NCERT Geography Class XI) CHAPTER-1

Q.60) Ans) c Exp) India, apart from many small islands, has two main archipelagos- Andaman and Nicobar Islands in the Bay of Bengal, and the Lakshadweep islands in Arabian sea. The Bay of Bengal island groups consist of about 572 islands/islets. These are situated roughly between 6°N-14°N and 92°E -94°E. The two principal groups of islets include the Ritchie’s archipelago and the Labyrinth island. The entire group of islands is divided into two broad categories – the Andaman in the north and the Nicobar in the South, separated by the Ten-degree channel. Hence, statement 1 is correct. It is believed that these islands are an elevated portion of submarine mountains.The only volcano in India, Barren Island, is located in Andaman and Nicobar. It is an active volcano and had last erupted in 2017. It also has a mud volcano situated in Baratang island, these mud volcanoes have erupted sporadically, with recent eruptions in 2005 believed to have been associated with the 2004 Indian Ocean earthquake. Statement 2 is correct.

ForumIAS Offline Guidance Centre 2nd Floor, IAPL House, 19, Pusa Road, Karol Bagh, New Delhi – 110005 | [email protected]|9821711605 https://t.me/UPSC_PDF www.upscpdf.com

PTS 2020|1|Test 4 - Solutions |ForumIAS

Islands of Lakshadweep in the Arabian sea are located off the coast of Kerala. The main archipelago is separated from Minicoy by the 9-degree channel. Minicoy in turn is separated from Maldives by the 8-degree channel. Lakshadweep islands are made of coral deposits and there are many atolls. Statement 3 and 4 are correct.

Source) India: Physical Environment (NCERT Geography Class XI) CHAPTER-1

Q.61) Ans) c Exp) Option (c) is the correct answer. Statement 1 is correct. Financial Action Task Force (FATF) consists of two types of list- Black List and Grey List. Only those countries are included in black list that FATF considers as uncooperative tax havens. These countries are known as Non-Cooperative Countries or Territories (NCCTs). These countries do not support terrorist funding and money laundering activities. Statement 2 is correct. Countries which are considered as the safe havens for supporting terror funding and money laundering are included in the grey list. These countries may face economic sanctions and problems in getting loans from international organizations like IMF, ADB, World Bank etc.

KB) ● Pakistan has been placed on the Financial Action Task Force (FATF) grey list for failing to curb terror funding, ● Pakistan’s recent action against terror financing, particularly on the legal front, was found to be unsatisfactory, according to a review by the Asia Pacific Policy Group (APPG). ● The placement on to grey list could hurt Pakistan’s economy as well as its international standing.

Source) https://indianexpress.com/article/pakistan/pakistan-action-against-terror-funding- not-satisfactory-fatf-body-5359545/

Q.62) Ans) b Exp) Duars are alluvial floodplains found in Darjeeling and Sikkim HImalayas.. They are the result of folding during the formation of the Himalayas. The moderate slope, thick soil cover with high organic content, uniform year-round rainfall with mild winters paved way for their development into tea plantations by the British.

ForumIAS Offline Guidance Centre 2nd Floor, IAPL House, 19, Pusa Road, Karol Bagh, New Delhi – 110005 | [email protected]|9821711605 https://t.me/UPSC_PDF www.upscpdf.com

PTS 2020|1|Test 4 - Solutions |ForumIAS

They are known for their scenic beauty and are dotted with tourist destinations such as Darjeeling and Siliguri. They are home to many endangered species like one horned rhino and tiger. National parks and wildlife sanctuaries are found in large numbers such as Manas, Jaldpara, Buxa National Parks and Mahananda Wildlife Sanctuary. The Shivalik Hills, also known as Churia Hills, are a mountain range of the outer Himalayas that stretches from the Indus River about 2,400 km (1,500 mi) eastwards close to the Brahmaputra River. the Shivalik Hills belong to the tertiary deposits of the outer Himalayas. They are chiefly composed of sandstone and conglomerate rock formations, which are the solidified detritus of the Himalayas to their north; they are poorly consolidated. The remanent magnetisation of siltstones and sandstones indicates that they were deposited 16–5.2 million years ago. They are the southernmost and geologically youngest east-west mountain chain of the Himalayas The term Karewa is derived from the Kashmiri dialect which means “elevated table-land”. Karewas are lacustrine deposits with a thickness of about 1400 m. It lies between the Pre Panjal Range and the Great Himalayan Range, in Northwest India. Geologists believe that region of Kashmir valley was filled with water of Tethys Sea but with the passage of time, many earthquakes came and a part of baramulla range broke down that resulted in emptying of water from the valley leaving behind the moulds of karewas. These deposits are composed of sand, silt, clay, shale, mud, lignite, gravel and loessic sediments. Therefore, it is extremely important for agricultural and horticultural practices in the valley. The world-famous variety of saffron, which is locally known as “zafran” is cultivated on this deposits. Kayals - The Kerala backwaters are a network of brackish lagoons and lakes lying parallel to the Arabian Sea coast (known as the Malabar Coast) of Kerala state in southern India, as well as interconnected canals, rivers, and inlets, a labyrinthine system formed by more than 900 kilometres (560 mi) of waterways. The backwaters were formed by the action of waves and shore currents creating low barrier islands across the mouths of the many rivers flowing down from the Western ghats range.

Source) India: Physical Environment (NCERT Geography Class XI) CHAPTER-2

Q.63) Ans) a Exp) Option (a) is the correct answer. This is the most widely spread and important soil. The alluvial soil consists of various proportions of sand, silt and clay. As we move inland towards the river valleys, soil particles appear somewhat bigger in size. In the upper reaches of the river valley i.e. near the place of the break of slope, the soils are coarse. Such soils are more common in piedmont plains such as Duars, Chos and Terai.

ForumIAS Offline Guidance Centre 2nd Floor, IAPL House, 19, Pusa Road, Karol Bagh, New Delhi – 110005 | [email protected]|9821711605 https://t.me/UPSC_PDF www.upscpdf.com

PTS 2020|1|Test 4 - Solutions |ForumIAS

Alluvial soils as a whole are very fertile. Mostly these soils contain adequate proportion of potash, phosphoric acid and lime which are ideal for the growth of sugarcane, paddy, wheat and other cereal and pulse crops. Due to its high fertility, regions of alluvial soils are intensively cultivated and densely populated. Soils in the drier areas are more alkaline and can be productive after proper treatment and irrigation. Statement 1 is correct. Alluvial soil is the most widely spread and important soil. In fact, the entire northern plains are made of alluvial soil. These have been deposited by three important Himalayan river systems– the Indus, the Ganga and the Brahmaputra. These soils also extend in Rajasthan and Gujarat through a narrow corridor. Alluvial soil is also found in the eastern coastal plains particularly in the deltas of the Mahanadi, Godavari, Krishna and Kaveri rivers. Statement 2 is incorrect. Soils are also described on the basis of their age. According to their age alluvial soils can be classified as old alluvial (Bangar) and new alluvial (Khadar). Statement 3 is incorrect. The Bangar soil has higher concentration of kanker nodules than the Khadar whereas Khadar has more fine particles and is more fertile than the Bangar.

Source) Chap-1, page no.-8, contemporary india-2

Q.64) Ans) b Exp) Statement 1 is correct. The Karbi Anglong-Meghalaya plateau is an extension of the Peninsular plateau. They are two different plateaus - Karbi Anglong Plateau and Meghalaya plateau. Statement 2 is incorrect. These are separated from the Peninsular plateau by a huge fault. This fault is believed to have formed due to the force exerted by the north-eastward movement of the Indian plate at the time of formation of the Himalayas. The fault was created between the Rajmahal hills and the Karbi-Meghalaya plateau. It later filled up with river deposits from numerous rivers. The plateau region receives most of its rainfall from the south west monsoons. Places like Cherrapunji and Mawsynram are located here, which receive the highest rainfall in the world. Statement 3 is correct. Despite heavy rainfall the region has an eroded and bare rocky structure, with little to no permanent vegetation.

Source) India: Physical Environment (NCERT Geography Class XI) CHAPTER-2

Q.65) Ans) b Exp) Option (b) is the correct answer.

ForumIAS Offline Guidance Centre 2nd Floor, IAPL House, 19, Pusa Road, Karol Bagh, New Delhi – 110005 | [email protected]|9821711605 https://t.me/UPSC_PDF www.upscpdf.com

PTS 2020|1|Test 4 - Solutions |ForumIAS

Statement 1 is correct. e-Sahaj’ portal has been launched to grant a Security Clearance by the Ministry of Home Affairs. It is set up to facilitate individuals and private companies in seeking security clearances for setting up businesses in certain sensitive sectors. The portal facilitates an applicant to submit the application online and also to view the status of his application from time to time. Statement 2 is incorrect. The introduction of online portal has made the process standardized, which is faster, transparent and easy to monitor. Various functionaries can access the application and documents online and take timely decisions. Statement 3 is correct. The objective of national security clearance is to evaluate potential security threats, including economic threats, and provide risk assessment before clearing investment and project proposals in key sectors. It aims to strike a healthy balance between meeting the imperatives of national security and facilitating ease of doing business and promoting investment in the country.

Source) http://pib.nic.in/newsite/PrintRelease.aspx?relid=183549

Q.66) Ans) b Exp) Indian terrain is interspersed with lakes, both natural and artificial. A few of these are also part of the Ramsar Convention on Wetlands. These lakes provide livelihood and are home to large number of faunal and avi-faunal variety. Deepor Beel in Assam is a Ramsar site and it harbors a wide variety of flora and fauna and also suffers from anthropological meddling and pollution. Hence, statement 1 is correct. Nal Sarovar in Gujarat is also known for its bird sanctuary. It was declared a Ramsar site in 2012 Statement 2 is incorrect. Sasthamkotta is the largest fresh water lake in Kerala. It is a Ramsar site. The lake provides drinking water to nearby areas, and the purity of water is attributed to the presence of large population of cavaborus the bacteria in lake water. Statement 3 is incorrect. Kanjli lake is an artificial lake in Punjab formed from the waters of Beas. It was recognized as a Ramsar site in 2002. Statement 4 is correct.

Source) India: Physical Environment (NCERT Geography Class XI) CHAPTER-2

Q.67) Ans) b Exp) Karewas: During the formation of Pir Panjal range large lakes were formed as rivers were impounded. These lakes continued to receive sediments from rivers. With time water drained

ForumIAS Offline Guidance Centre 2nd Floor, IAPL House, 19, Pusa Road, Karol Bagh, New Delhi – 110005 | [email protected]|9821711605 https://t.me/UPSC_PDF www.upscpdf.com

PTS 2020|1|Test 4 - Solutions |ForumIAS

away and thick deposits of glacial clay and other materials embedded with moraines were left behind. This resulted in formation of lacustrine plains known as Karewas. These are useful for cultivation of Zafran and are of great economic significance to the Kashmir Valley. Option 1 is correct. Kayals: These are backwaters or lagoons formed at the Malabar coast. They are used for fishing, inland navigation, boat racing events, and tourism. Option 2 is incorrect. Patland plateaus: These are lava capped plateaus which have been formed by continuous denudation of high plateaus. They have been reduced from high and extensive plateaus to isolated hills with flat tops with steep slopes. The Peninsular India is made up of series of such plateaus- Hazaribagh plateau, Palamu plateau, Ranchi plateau etc. Option 3 is correct. Mangroves: These are tropical coastal vegetation that grow along the coast in salt marshes, tidal creeks, mud flats and estuaries. These are halophytes, or salt tolerant and have adapted to harsh coastal conditions through salt filtration system, low oxygen requirement and viviparous seeds. In India they are found in Andaman and Nicobar, Suderbans of West Bengal, Mahanadi, Godavari, and Krishna deltas. Option 4 is correct. Badlands/Ravines: These are a type of fluvial erosion. They are formed as a result of constant vertical erosion by streams and rivers flowing over semi-arid and arid regions. The area through which Chambal flows does not receive enough rainfall and has created ravines 60–80m in depth. Option 5 is correct. Estuaries: These are the transition zones between river and marine environment. They have brackish water. They are different from deltas, as deltas are a depositional landmass, while estuaries are water bodies. In India they are usually formed on the Western coast. Option 6 is incorrect.

Source) India: Physical Environment (NCERT Geography Class XI) CHAPTER-2

Q.68) Ans) c Exp) Option (c) is the correct answer. Statement 1 is correct. Cyclotrons are used to produce radioisotopes for diagnostic and therapeutic use for cancer care. Cyclone-30, the biggest cyclotron in India for medical application became operational in September 2018. Statement 2 is correct. Cyclone-30 facility at VECC, Kolkata, a Department of Atomic Energy (DAE) unit, will have many unique features, under various stages of implementation, which are first of its kind in many ways. This facility will provide for affordable radioisotopes and related radiopharmaceuticals for the entire country especially for Eastern India for cancer treatment.

ForumIAS Offline Guidance Centre 2nd Floor, IAPL House, 19, Pusa Road, Karol Bagh, New Delhi – 110005 | [email protected]|9821711605 https://t.me/UPSC_PDF www.upscpdf.com

PTS 2020|1|Test 4 - Solutions |ForumIAS

It will also have in-situ production of Gallium-68 and Palladium-103 isotopes, used for breast cancer diagnosis and prostate cancer treatment, respectively.

Source) http://pib.nic.in/newsite/PrintRelease.aspx?relid=183536

Q.69) Ans) d Exp) All Statements are incorrect: India is the largest producer as well as the consumer of pulses in the world. These are the major source of protein in a vegetarian. diet. Major pulses that are grown in India are tur (arhar), urad, moong, masur, peas and gram. Pulses need less moisture and survive even in dry conditions. Being leguminous crops, all these crops except arhar help in restoring soil fertility by fixing nitrogen from the air. Therefore, these are mostly grown in rotation with other crops. Major pulses producing states in India are Madhya Pradesh, Uttar Pradesh, Rajasthan, Maharashtra and Karnataka.

Source) Contemporary India II (Class 10 NCERT) Chapter 4 : Agriculture Page no : 38,40

Q.70) Ans) d Exp) All statements are correct: The FCI procures food grains from the farmers at the government announced minimum support price (MSP). The government provides subsidies on agriculture inputs such as fertilizers, power and water. These subsidies have now reached unsustainable levels and have also led to large scale inefficiencies in the use of these scarce resources. Excessive and imprudent use of fertilizers and water has led to waterlogging, salinity and depletion of essential micronutrients in the soil. The high MSP leads to farmers becoming reluctant in selling their produce in the markets. This leads to a shortage of grains and thus fuels inflation in the economy. The committed FCI purchases also distorts the cropping pattern as such produce is kept in big godowns where it is prone to rotting. This also creates a shortage of supply in the market. The inefficiency in transportation and storage of grains leads to heavy losses both of the produce and economy. Example -Wheat and paddy crops are being grown more because of the MSP they get. Punjab and Haryana are foremost examples.

KB) Cropping Pattern means proportion of area under different crops at different points of time. Food Corporation of India (FCI) is set up under the Food Corporations Act 1964 and is responsible for procuring and stocking foodgrains, whereas distribution is ensured by public distribution system (PDS).

ForumIAS Offline Guidance Centre 2nd Floor, IAPL House, 19, Pusa Road, Karol Bagh, New Delhi – 110005 | [email protected]|9821711605 https://t.me/UPSC_PDF www.upscpdf.com

PTS 2020|1|Test 4 - Solutions |ForumIAS

Source) Contemporary India II (Class 10 NCERT) Chapter 4 : Agriculture Page no : 45

Q.71) Ans) a Exp) Option (a) is the correct answer. The Tropic of Cancer (23° 30'N) divides the country into two parts. It passes through 8 Indian states. These states are from west to east Gujarat, Rajasthan, Madhya Pradesh, Chhattisgarh, Jharkhand, West Bengal, Tripura and Mizoram.

Source) NCERT Class IX- Social Science, Contemporary India – 1 (Geography); Chapter-1 (India – Size and Location), Page No. 5 (India Map)

Q.72) Ans) d Exp) Statement 1 and 2 are correct: Tea cultivation is an example of plantation agriculture. The tea plant grows well in tropical and sub-tropical climates endowed with deep and fertile well-drained soil, rich in humus and organic matter. Tea bushes require warm and moist frost- free climate all through the year. Frequent showers evenly distributed over the year ensure continuous growth of tender leaves.

Statement 3 is incorrect: Major tea producing states are Assam, hills of Darjeeling and Jalpaiguri districts, West Bengal, Tamil Nadu and Kerala. Apart from these, Himachal Pradesh, Uttaranchal, Meghalaya, Andhra Pradesh and Tripura are also tea-producing states in the country.

ForumIAS Offline Guidance Centre 2nd Floor, IAPL House, 19, Pusa Road, Karol Bagh, New Delhi – 110005 | [email protected]|9821711605 https://t.me/UPSC_PDF www.upscpdf.com

PTS 2020|1|Test 4 - Solutions |ForumIAS

Source) Contemporary India II (Class 10 NCERT) Chapter 4 : Agriculture Page no : 40,41

Q.73) Ans) b Exp) Option (b) is the correct answer. Statement 1 is incorrect. The National Database on Sexual Offenders (NDSO) is a central database of sexual offenders in the country which is maintained by the NCRB for regular monitoring and tracking by the State Police. The database is accessible only to the law enforcement agencies (not common people) for investigation and monitoring purpose. The database will include offenders convicted under charges of rape, gang rape, POCSO and eve teasing. Statement 2 is correct. The Cyber Crime Prevention Against Women and Children (CCPWC) portal is convenient and user friendly that enables complainants in reporting cases without disclosing their identity. This not only helps the victims/complainants but also help the civil society organizations and responsible citizens to anonymously report complaints pertaining to child pornography, child sexual abuse material or sexually explicit material. Complainants can also upload the objectionable content and URL to assist in the investigation by the State Police. The complaints registered through this portal will be handled by police authorities of respective State/UTs. A victim or complainant can track his/her report by opting for “report and track” option using his/her mobile number. Statement 3 is correct. NCRB was set-up in 1986 to function as a repository of information on crime and criminals so as to assist the investigators in linking crime to the perpetrators. It was entrusted with the responsibility for monitoring, coordinating and implementing the Crime and Criminal Tracking Network & Systems (CCTNS) project in the year 2009. The project connects 15000+ police stations and 6000 higher offices of police in the country.The National Crime Records Bureau (NCRB) proactively identifies sexually objectionable content and takes it up with intermediaries for its removal.

Source) http://pib.nic.in/newsite/PrintRelease.aspx?relid=183597

Q.74) Ans) a Exp) All the statements are correct: Minerals found in India are unevenly distributed. Broadly, peninsular rocks contain most of the reserves of coal, metallic minerals, mica and many other non-metallic minerals. Sedimentary rocks on the western and eastern flanks of the peninsula, in Gujarat and Assam have most of the petroleum deposits. Rajasthan with the rock systems of the peninsula, has reserves of many non-ferrous minerals. The vast alluvial

ForumIAS Offline Guidance Centre 2nd Floor, IAPL House, 19, Pusa Road, Karol Bagh, New Delhi – 110005 | [email protected]|9821711605 https://t.me/UPSC_PDF www.upscpdf.com

PTS 2020|1|Test 4 - Solutions |ForumIAS

plains of north India are almost devoid of economic minerals. These variations exist largely because of the differences in the geological structure, processes and time involved in the formation of minerals. India is critically deficient in the reserve and production of copper. The Balaghat mines in Madhya Pradesh produce 52 per cent of India’s copper. The Singhbhum district of Jharkhand is also a leading producer of copper. The Khetri mines in Rajasthan are also famous.

Source) Contemporary India II (Class 10 NCERT) Chapter 5 : MINERALS AND ENERGY RESOURCES Page no : 52

Q.75) Ans) d Exp) Minerals are usually found in “ores”. The term ore is used to describe an accumulation of any mineral mixed with other elements. The mineral content of the ore must be in sufficient concentration to make its extraction commercially viable. The type of formation or structure in which they are found determines the relative ease with which mineral ores may be mined. This also determines the cost of extraction. It is, therefore, important for us to understand the main types of formations in which minerals occur. Minerals generally occur in these forms: (i) In igneous and metamorphic rocks minerals may occur in the cracks, crevices, faults or joints. The smaller occurrences are called veins and the larger are called lodes. Thus statement 1 is incorrect. In sedimentary rocks a number of minerals occur in beds or layers. They have been formed as a result of deposition, accumulation and concentration in horizontal strata. Thus statement 2 is incorrect.

Source) Contemporary India II (Class 10 NCERT) Chapter 5 : MINERALS AND ENERGY RESOURCES Page no : 51

Q.76) Ans) c Exp) Option (c) is the correct answer. Statement 1 is not correct. Nepal shares a border with 5 Indian states namely Uttarakhand, Uttar Pradesh, Bihar, West Bengal and Sikkim. Assam does not have border with Nepal. Statement 2 is correct. The Longitude 76o0’E passes through 9 states namely Jammu & Kashmir, Himachal Pradesh, Punjab, Haryana, Rajasthan, Madhya Pradesh, Maharashtra, Karnataka and Kerala.

ForumIAS Offline Guidance Centre 2nd Floor, IAPL House, 19, Pusa Road, Karol Bagh, New Delhi – 110005 | [email protected]|9821711605 https://t.me/UPSC_PDF www.upscpdf.com

PTS 2020|1|Test 4 - Solutions |ForumIAS

Statement 3 is not correct. Maharashtra has border with 6 Indian states namely Gujarat, Madhya Pradesh, Chhattisgarh, Telangana, Karnataka and Goa.

ForumIAS Offline Guidance Centre 2nd Floor, IAPL House, 19, Pusa Road, Karol Bagh, New Delhi – 110005 | [email protected]|9821711605 https://t.me/UPSC_PDF www.upscpdf.com

PTS 2020|1|Test 4 - Solutions |ForumIAS

Q.77) Ans) a Exp) Option 1 is correct: India’s trade with foreign countries is carried from the ports located along the coast. Approximately 95 per cent of the country’s trade volume (68 per cent in terms of value) is moved by sea. Option 2 is incorrect: The Brahmaputra river between Sadiya and Dhubri has been declared as National Waterway 2. Option 3 is incorrect: Paradip port is located in Odisha and Visakhapatnam port is located in Andhra Pradesh. Both the ports specialize in export of iron ore.

KB) Apart from these, Other National waterways are - National Waterway-1: Allahabad-Haldia stretch of the Ganga-Bhagirathi-Hooghly river in the states of Uttar Pradesh, Bihar, Jharkhand and West Bengal National Waterway-3: Kollam-Kottapuram stretch of West Coast Canal and Champakara and Udyogamandal canals in the state of Kerala National Waterway-4: In the states of Andhra Pradesh and Tamil Nadu and the Union Territory of Puducherry

ForumIAS Offline Guidance Centre 2nd Floor, IAPL House, 19, Pusa Road, Karol Bagh, New Delhi – 110005 | [email protected]|9821711605 https://t.me/UPSC_PDF www.upscpdf.com

PTS 2020|1|Test 4 - Solutions |ForumIAS

National Waterway-5: In the states of West Bengal and Odisha National Waterways 6: National Waterways 6 is a proposed waterway between Lakhipur and Bhanga of the Barak River with a length of 121 km.

Q.78) Ans) c Exp) Option (c) is the correct answer. Statement 1 is correct. Coral polyps are short-lived microorganisms, which live in colonies. They flourish in shallow, mud free and warm waters. They secrete calcium carbonate. They are mainly of three kinds: Barrier reef, Fringing reef and Atolls. All 3 are found in India. Statement 2 is not correct. Coral reefs are present in the areas of Gulf of Kutch, Gulf of Mannar, Andaman & Nicobar, Lakshadweep Islands and Malvan (Maharashtra). Statement 3 is correct. Coral polyps live in a Symbiotic relationship with algae, crucial for the health of the coral and the reef. The microscopic algae capture sunlight and convert it into energy, to provide essential nutrients to the corals. In exchange, they have a place to live inside the animal's body. When coral polyps expel algae that live inside their tissues, Coral bleaching occurs.

KB) The conditions best suited for building of coral reefs are: 1) The minimum water temperature required is 20o C. Hence, distribution of corals is mainly limited to tropical and subtropical zones. The Pacific and Indian Oceans have the most coral reefs. 2) The depth of water should not exceed 180 feet, because beyond this depth sunlight is weak for photosynthesis to occur. This is essential for survival of algae, on which coral polyps depends. Shallow water of less than 100 feet is thus ideal. But there should be plenty of water as polyps cannot survive for too long out of water. 3) The water should be saltish and free from sediments. Coral best survive in moving ocean water. Corals are best developed on the seaward side of the reef, where constantly moving waves, tides & currents maintain abundant supply of clear, oxygenated water. They also bring adequate amount of microorganisms which serves as food for polyps.

Source) NCERT Class IX- Social Science, Contemporary India – 1 (Geography); Chapter-2 (Physical Features of India), Page No. 15

Q.79) Ans) b Exp) Option (b) is the correct answer.

ForumIAS Offline Guidance Centre 2nd Floor, IAPL House, 19, Pusa Road, Karol Bagh, New Delhi – 110005 | [email protected]|9821711605 https://t.me/UPSC_PDF www.upscpdf.com

PTS 2020|1|Test 4 - Solutions |ForumIAS

Statement 1 is not correct. Gondwanaland is the southern part of the ancient supercontinent Pangea with AngaraLand in the northern part.The oldest landmass, (the Peninsula part of India), was a part of the Gondwana land. Statement 2 is correct. The Gondwanaland included India, Australia, South Africa, South America and Antarctica as one single land mass.The convection currents split the crust into a number of pieces, thus leading to the drifting of the Indo-Australian plate after being separated from the Gondwana land, towards north. The northward drift resulted in the collision of the plate with the much larger Eurasian Plate. Due to this collision, the sedimentary rocks which were accumulated in the geosyncline known as the Tethys were folded to form the mountain system of western Asia and Himalaya.

Source) NCERT Class IX- Social Science, Contemporary India – 1 (Geography); Chapter-2 (Physical Features of India), Page No. 8

Q.80) Ans) a Exp) Option (a) is the correct answer. Majuli is a river island in the Brahmaputra River, Assam and in 2016 it became the first island to be made a district in India. Majuli has shrunk as the river surrounding it has grown. Majuli is currently listed as the world's largest river island in the Guinness Book of World Records. The island has been the hub of Assamese neo-Vaishnavite culture, initiated around the 15th century by the revered Assamese saint Srimanta Sankardeva and his disciple Madhavdeva. Many ‘Satras’ or monasteries constructed by the saint still survive and represent the colourful Assamese culture The Mising (or Miris) community has the largest population in the island. They are also found in the state of Arunachal Pradesh. Misings are recognised as a Scheduled Tribe by the Indian government under the name 'Miri'.

Source): NCERT Class IX- Social Science, Contemporary India – 1 (Geography); Chapter-2 (Physical Features of India), Page No. 12

Q.81) Ans) b Exp) Option (b) is the correct answer.

ForumIAS Offline Guidance Centre 2nd Floor, IAPL House, 19, Pusa Road, Karol Bagh, New Delhi – 110005 | [email protected]|9821711605 https://t.me/UPSC_PDF www.upscpdf.com

PTS 2020|1|Test 4 - Solutions |ForumIAS

Statement 1 is correct. The radial pattern develops when streams flow in different directions from a central peak or dome-like structure.

The streams within a drainage basin form certain patterns, depending on the 3 major factors: 1) Slope of land 2) Underlying rock structure 3) Climatic conditions of the area The 4 major drainage patterns are- Dendritic, Trellis, Rectangular, Radial. Statement 2 is not correct. The dendritic pattern develops where the river channel follows the slope of the terrain. The stream with its tributaries resembles the branches of a tree, thus the name dendritic. Statement 3 is not correct. A river joined by its tributaries, at approximately right angles, develops a trellis pattern. A trellis drainage pattern develops where hard and soft rocks exist parallel to each other.

KB) Rectangular Pattern: A rectangular drainage pattern develops on a strongly jointed rocky terrain. When the rivers discharge their waters from all directions in a lake or depression, the pattern is known as ‘centripetal’.

Source) NCERT Class IX- Social Science, Contemporary India – 1 (Geography); Chapter-3 (Drainage), Page No. 18

ForumIAS Offline Guidance Centre 2nd Floor, IAPL House, 19, Pusa Road, Karol Bagh, New Delhi – 110005 | [email protected]|9821711605 https://t.me/UPSC_PDF www.upscpdf.com

PTS 2020|1|Test 4 - Solutions |ForumIAS

Q.82) Ans) d Exp) Option (d) is the correct answer. Option A is not the correct answer. Naramada rises in Amarkantak hills, Madhya Pradesh. It flows towards the west in a rift valley formed due to faulting. Its basin covers parts of Madhya Pradesh, Gujarat. Option B is not the correct answer. Sabarmati River is one of the major west-flowing rivers in India. It originates in the Aravalli Range, Rajasthan and meets the Arabian Sea. Its drainage basin covers Rajasthan and Gujarat. Option C is not the correct answer. Mahanadi is an east flowing river and drains into Bay of Bengal. It rises in the highlands of Chhattisgarh. Its drainage basin covers states of Maharashtra, Chhattisgarh, Jharkhand and Odisha. Option D is the correct answer. The Tapi River rises in Satpura ranges in Betul district of Madhya Pradesh. It is a west flowing river and drains in Arabian Sea. It flows in a rift valley parallel to Narmada. Its basin covers parts of Madhya Pradesh, Gujarat and Maharashtra.

KB) Other major west flowing rivers are: Mahi, Bharathpuzha, Periyar.

Source) NCERT Class IX- Social Science, Contemporary India – 1 (Geography); Chapter-3 (Drainage), Page No. 21

Q.83) Ans) c Exp) Option (c) is the correct answer. There are differences of opinion about the evolution of the Himalayan rivers. However, geologists believe that a mighty river called Shiwalik or Indo-Brahma traversed the entire longitudinal extent of the Himalaya from Assam to Punjab and onwards to Sind, and finally discharged into the Gulf of Sind near lower Punjab during the Miocene period some 5-24 million years ago. The remarkable continuity of the Shiwalik and its lacustrine origin and alluvial deposits consisting of sand, silt, clay, boulders and conglomerates support this viewpoint. It is opined that in due course of time Indo– Brahma river was dismembered into three main drainage systems: (i) the Indus and its five tributaries in the western part; (ii) the Ganga and its Himalayan tributaries in the central part; and (iii) the stretch of the Brahmaputra in Assam and its Himalayan tributaries in the eastern part. Statement 1 is correct. The dismemberment was probably due to the Pleistocene upheaval in the western Himalayas, including the uplift of the Potwar Plateau (Delhi Ridge), which acted as the water divide between the Indus and Ganga drainage systems. Statement 2 is also correct. The downthrusting of the Malda gap area between the

ForumIAS Offline Guidance Centre 2nd Floor, IAPL House, 19, Pusa Road, Karol Bagh, New Delhi – 110005 | [email protected]|9821711605 https://t.me/UPSC_PDF www.upscpdf.com

PTS 2020|1|Test 4 - Solutions |ForumIAS

Rajmahal hills and the Meghalaya plateau during the mid-pleistocene period, diverted the Ganga and the Brahmaputra systems to flow towards the Bay of Bengal.

Source) NCERT Class XI; Chapter-3 (Drainage), Page No. 24

Q.84) Ans) c Exp) Option (c) is the correct answer. Statement 1 is correct. Kolleru Lake is one of the largest freshwater lakes in India in state of Andhra Pradesh. Kolleru is located between Krishna and Godavari river deltas. It is designated a wetland of international importance in November 2002 under the international Ramsar Convention. Statement 2 is not correct. Nagarjuna Sagar reservoir lies on river Krishna whereas Krishnaraja Sagar reservoir is on river Kaveri. Statement 3 is not correct. The river Ganga is joined by many tributaries from the Himalayas such as Yamuna, Ghaghra, Gandak, Kosi. The river Yamuna rises from Yamunotri glacier flows parallel to the river Ganga as a right bank tributary. It meets Ganga at Allahabad.

ForumIAS Offline Guidance Centre 2nd Floor, IAPL House, 19, Pusa Road, Karol Bagh, New Delhi – 110005 | [email protected]|9821711605 https://t.me/UPSC_PDF www.upscpdf.com

PTS 2020|1|Test 4 - Solutions |ForumIAS

Source) NCERT Class IX- Social Science, Contemporary India – 1 (Geography); Chapter-3 (Drainage), Page No. 19, 20

Q.85) Ans) b Exp) Option (b) is the correct answer. Statement 1 is incorrect. Endangered Species: These are species which are in danger of extinction. The survival of such species is difficult if the negative factors that have led to a decline in their population continue to operate. The examples of such species are black buck, crocodile, Indian wild ass, Indian rhino, lion tailed macaque, sangai (brow anter deer in Manipur), etc. Statement 2 is incorrect. Endemic Species: These are species which are only found in some particular areas usually isolated by natural or geographical barriers. Examples of such species are the Andaman teal, Nicobar pigeon, Andaman wild pig, mithun in Arunachal Pradesh. Statement 3 is correct. Extinct Species: These are species which are not found after searches of known or likely areas where they may occur. A species may be extinct from a local area, region, country, continent or the entire earth. Examples of such species are the Asiatic cheetah, pink head duck.

KB) Vulnerable Species: These are species whose population has declined to levels from where it is likely to move into the endangered category in the near future if the negative factors continue to operate. The examples of such species are blue sheep, Asiatic elephant, Gangetic dolphin, etc. Rare Species: Species with small population may move into the endangered or vulnerable category if the negative factors affecting them continue to operate. The examples of such species are the Himalayan brown bear, wild Asiatic buffalo, desert fox and hornbill, etc.

Source) Chap-2, page no- 15, contemporary india -2

Q.86) Ans) d Exp) Option (d) is the correct answer. Statement 1 and 2 is correct. Sacred groves - a wealth of diverse and rare species. Nature worship is an age-old tribal belief based on the premise that all creations of nature have to be protected. Such beliefs have preserved several virgin forests in pristine form called Sacred Groves (the forests of God and Goddesses). These patches of forest or parts of large forests have been left untouched by the local people and any interference with them is banned. Statement 3 is correct. The Mundas and the Santhal of Chhota Nagpur region worship mahua (Bassia latifolia) and kadamba (Anthocephalus cadamba) trees, and the tribals of Orissa and

ForumIAS Offline Guidance Centre 2nd Floor, IAPL House, 19, Pusa Road, Karol Bagh, New Delhi – 110005 | [email protected]|9821711605 https://t.me/UPSC_PDF www.upscpdf.com

PTS 2020|1|Test 4 - Solutions |ForumIAS

Bihar worship the tamarind (Tamarindus indica) and mango (Mangifera indica) trees during weddings. To many of us, peepal and banyan trees are considered sacred.

Source) Chap-3, Page No- 21, Contemporary India- 2 Feature)

Q.87) Ans) d Exp) Option (d) is the correct answer. Statement 1 is incorrect. Intensive Subsistence Farming This type of farming is practised in areas of high population pressure on land. It is labour intensive farming, where high doses of biochemical inputs and irrigation are used for obtaining higher production. Statement 2 is incorrect. Primitive Subsistence Farming This type of farming is still practised in few pockets of India. Primitive subsistence agriculture is practised on small patches of land with the help of primitive tools like hoe, dao and digging sticks, and family/community labour. This type of farming depends upon monsoon, natural fertility of the soil and suitability of other environmental conditions to the crops grown.

Source) Chap-4, Page no- 34&35

Q.88) Ans) a Exp) Option (a) is the correct answer. Statement 1 is correct. Rabi crops are sown in winter from October to December and harvested in summer from April to June. Some of the important rabi crops are wheat, barley, peas, gram and mustard. Though, these crops are grown in large parts of India, states from the north and northwestern parts such as Punjab, Haryana, Himachal Pradesh, Jammu and Kashmir, Uttaranchal and Uttar Pradesh are important for the production of wheat and other rabi crops. Availability of precipitation during winter months due to the western temperate cyclones helps in the success of these crops. However, the success of the green revolution in Punjab, Haryana, western Uttar Pradesh and parts of Rajasthan has also been an important factor in the growth of the abovementioned rabi crops. Statement 2 is incorrect. Kharif crops are grown with the onset of monsoon in different parts of the country and these are harvested in September-October. Important crops grown during this season are paddy, maize, jowar, bajra, tur (arhar), moong, urad, cotton, jute, groundnut and soyabean. Some of the most important rice-growing regions are Assam, West Bengal, coastal regions of Orissa, Andhra Pradesh, Tamil Nadu, Kerala and Maharashtra,

ForumIAS Offline Guidance Centre 2nd Floor, IAPL House, 19, Pusa Road, Karol Bagh, New Delhi – 110005 | [email protected]|9821711605 https://t.me/UPSC_PDF www.upscpdf.com

PTS 2020|1|Test 4 - Solutions |ForumIAS

particularly the (Konkan coast) along with Uttar Pradesh and Bihar. Recently, paddy has also become an important crop of Punjab and Haryana. In states like Assam, West Bengal and Orissa, three crops of paddy are grown in a year. These are Aus, Aman and Boro. Statement 3 is incorrect. In between the rabi and the kharif seasons, there is a short season during the summer months known as the Zaid season. Some of the crops produced during ‘zaid’ are watermelon, muskmelon, cucumber, vegetables and fodder crops.

Source) Chap-4, page no- 36, contemporary India-2

Q.89) Ans) d Exp) Both the statements are wrong. Statement 1 is wrong. A gulf is a deep inlet of the sea with a narrow mouth, which is almost surrounded by land. Gulf can be defined as a portion of the sea that penetrates the land. Gulfs over the world can vary extensively in depth, size, and shape. Gulf is sometimes also known as a large bay. However, a bay is always larger and deeply indented than a gulf. Bays, as well as gulfs, make excellent trading centers and harbors due to their shape. The Gulf of Mexico (largest gulf in the world), Gulf of California, Persian Gulf, and the Gulf of Aden are some famous of gulfs. Statement 2 is wrong. A strait is a naturally formed narrow waterway that connects two large bodies of water. A strait can be formed by tectonic shifts or land that has been subsided or been eroded. The Strait of Gibraltar, the link between the Mediterranean Sea and the Atlantic Ocean was formed by tectonic shift. The Bosporus, which connects the Aegean Sea and the Black Sea, is believed to be formed by land that has subsided or eroded. If a strait is formed by human activities, it is called a canal. The Suez Canal, which links the Meditation Sea and the Red Sea, enables easy passage between Europe and Asia, is such a man- made strait. Gulfs can sometimes be connected to the ocean by straits.

Source) Class XI NCERT, Chapter 1, Page no. 5

Q.90) Ans) c Exp) The Mekong-Ganga Cooperation (MGC) is an initiative by six countries – India and five ASEAN countries, namely, Cambodia, Lao PDR, Myanmar, Thailand and Vietnam for cooperation in tourism, culture, education, as well as transport and communications. It was launched in 2000 at Vientiane, Lao PDR. Both the Ganga and the Mekong are civilizational rivers, and the

ForumIAS Offline Guidance Centre 2nd Floor, IAPL House, 19, Pusa Road, Karol Bagh, New Delhi – 110005 | [email protected]|9821711605 https://t.me/UPSC_PDF www.upscpdf.com

PTS 2020|1|Test 4 - Solutions |ForumIAS

MGC initiative aims to facilitate closer contacts among the people inhabiting these two major river basins. The MGC is also indicative of the cultural and commercial linkages among the member countries of the MGC down the centuries.

KB) The Mekong is a trans-boundary river in SouthEast Asia. From the Tibetan Plateau the river runs through China’s Yunnan Province, Myanmar, Laos, Thailand, Cambodia, and Vietnam. In 1995, Laos, Thailand, Cambodia, and Vietnam established the Mekong River Commission (MRC) to manage and coordinate the use and care of the Mekong.

Source) http://www.newsonair.com/Main-News-Details.aspx?id=369550

Q.91) Ans) c Exp) Option (c) is the correct answer. Statement 1 is correct. The Peninsular drainage system is older than the Himalayan one. This is evident from the broad, largely-graded shallow valleys, and the maturity of the rivers. Statement 3 is incorrect. Peninsular rivers are characterized by fixed course, absence of meanders and non-perennial flow of water. Absence of meanders due to the presence of hard peninsular rocks that are very difficult to erode. Non-perennial flow is due to the absence of snow-covered peaks and glaciers that provide water throughout the year. These rivers are only dependent on rainfall and thus have seasonal flow. The Himalayan drainage system has evolved through a long geological history. It mainly includes the Ganga, the Indus and the Brahmaputra river basins. Since these are fed both by melting of snow and precipitation, rivers of this system are perennial. These rivers pass through the giant gorges carved out by the erosional activity carried on simultaneously with the uplift of the Himalayas. Besides deep gorges, these rivers also form V-shaped valleys, rapids and waterfalls in their mountainous course. While entering the plains, they form depositional features like flat valleys, ox-bow lakes, flood plains, braided channels, and deltas near the river mouth. Statement 2 is correct. In the Himalayan reaches, the course of these rivers is highly tortuous, but over the plains they display a strong meandering tendency and shift their courses frequently.

Source) Class XI NCERT, Chapter 3, Page no. – 24

Q.92) Ans) a Exp) Option (a) is the correct answer. All the three types of forests are found in Andaman and Nicobar Islands.

ForumIAS Offline Guidance Centre 2nd Floor, IAPL House, 19, Pusa Road, Karol Bagh, New Delhi – 110005 | [email protected]|9821711605 https://t.me/UPSC_PDF www.upscpdf.com

PTS 2020|1|Test 4 - Solutions |ForumIAS

Source) http://pib.nic.in/newsite/PrintRelease.aspx?relid=67525 http://fsi.nic.in/isfr2017/isfr-mangrove-cover-2017.pdf

Q.93) Ans) d Exp) Statement 1 is correct. A mangrove is a shrub or small tree that grows in coastal saline or brackish water. The term is also used for tropical coastal vegetation consisting of such species. Mangroves occur worldwide in the tropics and subtropics, mainly between latitudes 25° N and 25° S. They require warm and wet conditions to survive and therefore cannot exist in temperate regions. Mangroves are salt-tolerant trees, also called halophytes, and are adapted to life in harsh coastal conditions. They contain a complex salt filtration system and complex root system to cope with salt water immersion and wave action. Statement 2 is correct. They are adapted to the low oxygen conditions of waterlogged mud, this adaptation leads to specialized root-like structures which stick up out of the soil like straws for breathing, known as pneumatophores. Statement 3 is also correct. Sundarbans in West Bengal account for almost half of the total area under mangroves in India.

KB) Some other adaptations by Mangroves: Because the soil is perpetually waterlogged, little free oxygen is available. Pneumatophores (aerial roots) allow mangroves to absorb gases directly from the atmosphere, and other nutrients such as iron, from the inhospitable soil. Mangroves store gases directly inside the roots, processing them even when the roots are submerged during high tide. A halophyte is a salt-tolerant plant that grows in soil or waters of high salinity, coming into contact with saline water through its roots or by salt spray, such as in saline semi-deserts, mangrove swamps, marshes and sloughs and seashores. Mangroves exclude salt by having significantly impermeable roots acting as an ultra-filtration mechanism to exclude sodium salts from the rest of the plant. Analysis of water inside mangroves has shown 90% to 97% of salt has been excluded at the roots. Because of the limited fresh water available in salty intertidal soils, mangroves limit the amount of water they lose through their leaves. They can restrict the opening of their stomata. They also vary the orientation of their leaves to avoid the harsh midday sun and so reduce evaporation from the leaves In this harsh environment, mangroves have evolved a special mechanism to help their offspring survive. Mangrove seeds are buoyant and are therefore suited to water dispersal. Unlike most plants, whose seeds germinate in soil, many mangroves (e.g. red mangrove) are viviparous, whose seeds germinate while still attached to the parent tree. Once germinated, the seedling grows

ForumIAS Offline Guidance Centre 2nd Floor, IAPL House, 19, Pusa Road, Karol Bagh, New Delhi – 110005 | [email protected]|9821711605 https://t.me/UPSC_PDF www.upscpdf.com

PTS 2020|1|Test 4 - Solutions |ForumIAS

either within the fruit or out through the fruit to form a propagule (a ready-to-go seedling) which can produce its own food via photosynthesis.

Source) http://fsi.nic.in/isfr2017/isfr-mangrove-cover-2017.pdf Class XI NCERT, Chapter 5, Page no. 61

Q.94) Ans) d Exp) The Himalayan Yew (Taxus wallachiana) is a medicinal plant found in various parts of Himachal Pradesh and Arunachal Pradesh. A chemical compound called ‘taxol’ is extracted from the bark, needles, twigs and roots of this tree, and it has been successfully used to treat some cancers – the drug is now the biggest selling anti-cancer drug in the world. The species is under great threat due to over-exploitation. In the last one decade, thousands of yew trees have dried up in various parts of Himachal Pradesh and Arunachal Pradesh. Source) Class X NCERT, Chapter 2, page no. – 17.

Q.95) Ans) d Exp) Option (d) is the correct answer. Statement 1 is correct. PCR will address issues such as information asymmetry, improve access to credit and strengthen the credit culture among consumers. Credit information is now available across multiple systems in bits and pieces and not in one window. Data on borrowings from banks, non-banking financial companies, corporate bonds or debentures from the market, external commercial borrowings (ECBs), foreign currency convertible bonds (FCCBs), masala bonds, and inter-corporate borrowings are not available in one data repository. Statement 2 is correct. Access to credit information, including debt details and repayment history would drive innovation in lending. For example, currently most banks focus on large companies for loans and consequently the micro, small and medium enterprises are left with limited options for borrowing. With satisfactory payment history and validated debt details made available, it will increase the credit availability to micro, small and medium enterprises along with deepening of the financial markets. This will support the policy of financial inclusion. Statement 3 is correct.

ForumIAS Offline Guidance Centre 2nd Floor, IAPL House, 19, Pusa Road, Karol Bagh, New Delhi – 110005 | [email protected]|9821711605 https://t.me/UPSC_PDF www.upscpdf.com

PTS 2020|1|Test 4 - Solutions |ForumIAS

PCR will help capture all relevant information about a borrower, across different borrowing products in one place. It can flag early warnings on asset quality by tracking performance on other credits and thus, strengthen credit culture among consumers. Statement 4 is correct. It can also address the bad loan problem staring at banks, as corporate debtors will not be able to borrow across banks without disclosing existing debt. A PCR may also help raise India’s rank in the global ease of doing business index.

KB) A public credit registry is an information repository which collates all loan information of individuals and corporate borrowers. A credit repository helps banks distinguish between a bad and a good borrower and accordingly offer attractive interest rates to good borrowers and higher interest rates to bad borrowers. PCR is based on the recommendations of a committee, headed by Y.M. Deosthalee.

Source) https://www.thehindu.com/business/why-is-a-public-credit-registry- important/article248

Q.96) Ans) d Exp) Gharial derives its name from ghara, an Indian word for pot because of a bulbous knob (narial excrescence) present at the end of their snout. The ghara also renders Gharial the only visibly sexually dimorphic crocodilian. The species are largely piscivorous of all extant crocodilians. Statement 2 is correct. Gharial prefers deep fast flowing rivers; however adult Gharial have also been observed in still water branches (jheel) of rivers and in comparatively velocity-free aquatic environments of deep holes (kunds) at river bends and confluences. Sand and rock outcrops are preferred basking sites and these animals show considerable site fidelity. Historically, Gharial were found in the river system of India, Pakistan, Bangladesh and southern part of Bhutan and Nepal. Today they survive only in the waters of India and Nepal. Statement 3 is also correct. The surviving population can be found within the tributaries of the Ganges river system: Girwa (Uttar Pradesh), Son (Madhya Pradesh), Ramganga (Uttarakhand), Gandak (Bihar), Chambal (Uttar Pradesh, Madhya Pradesh and Rajasthan) and Mahanadi (Orissa). Statement 1 is correct. It is currently listed as ‘Critically endangered’ under the IUCN category.

ForumIAS Offline Guidance Centre 2nd Floor, IAPL House, 19, Pusa Road, Karol Bagh, New Delhi – 110005 | [email protected]|9821711605 https://t.me/UPSC_PDF www.upscpdf.com

PTS 2020|1|Test 4 - Solutions |ForumIAS

KB) Sexual Dimorphism - It is the condition where the two sexes of the same species exhibit different characteristics beyond the differences in their sexual organs. The condition occurs in many animals and some plants. Differences may include secondary sex characteristics, size, weight, color, markings, and may also include behavioral and cognitive differences. Piscivorous - A piscivore is a carnivorous animal that eats primarily fish.

Source) Class X NCERT, Chapter 2, page no. – 18. https://www.wwfindia.org/about_wwf/priority_species/threatened_species/gharial/

Q.97) Ans) b Exp) Option (b) is the correct answer. NASA’s PMC Turbo mission launched a giant balloon on July 8th 2018 to study PMCs at a height of 50 miles above the surface. For five days, the balloon floated through the stratosphere from its launch at Esrange, Sweden, across the Arctic to Western Nunavut, Canada. This mission recently captured the images of noctilucent clouds or polar mesospheric clouds (PMCs). Noctilucent clouds coalesce as ice crystals on tiny meteor remnants in the upper atmosphere. These clouds are affected by atmospheric gravity waves caused by the convecting and uplifting of air masses, such as when air is pushed up by mountain ranges. The waves play major roles in transferring energy from the lower atmosphere to the mesosphere. These pictures might help scientists better understand turbulence in the atmosphere, as well as in oceans, lakes and other planetary atmospheres. This will lead to improved weather forecasting.

Source) https://indianexpress.com/article/technology/science/nasa-balloon-mission-may- help-improve-weather-forecasting-5368930/

Q.98) Ans) d Exp) A cryptocurrency is a tradable digital asset or digital form of money, built on blockchain technology that only exists online. Cryptocurrencies use cryptography to verify and secure transactions, hence their name. There are currently well over one thousand different cryptocurrencies in the world and many people see them as the lynchpin of a fairer, future economy. Statement 1 is incorrect. One of the biggest draws of cryptocurrency and even the blockchain, in general, is its leaning towards decentralization. While it can be applied to many different things, the main idea involving all decentralized systems is that they can’t be controlled by any one authority. This makes them more democratic in nature, and it means that you don’t need to

ForumIAS Offline Guidance Centre 2nd Floor, IAPL House, 19, Pusa Road, Karol Bagh, New Delhi – 110005 | [email protected]|9821711605 https://t.me/UPSC_PDF www.upscpdf.com

PTS 2020|1|Test 4 - Solutions |ForumIAS

worry about one entity being able to make off with your funds or that the system will be shut down by outside interference. Statement 2 is incorrect. On February 26, 2018 the Republic of the Marshall Islands issued a new blockchain based currency called Sovereign (‘SOV’) as a “legal tender of Marshall Islands for all debts, public charges, taxes and dues.” By virtue of being legal tender of Marshall Islands, SOV is legally defined as money worldwide. KB) To date, countries that have issued their own cryptocurrencies include Ecuador, China, Senegal, Singapore, Tunisia, though these countries will not be standing alone for long with Estonia, Japan, Palestine, Russia and Sweden looking to launch their own national cryptocurrencies. Some of these countries are likely to take it a step further and replace paper tender altogether with China being one nation that is looking to take one step beyond a virtual and paper version. Some national cryptocurrencies are: Tunisia - eCFA, Marshall Islands - Sovereign, Senegal - eCFA, Venezuela Petro.

Source) https://indianexpress.com/article/explained/understanding-cryptocurrencies-whats- to-like-and-whats-to-fear-5859083/

Q.99) Ans) d Exp) The Regional Comprehensive Economic Partnership (RCEP) is a so-called mega-regional economic agreement being negotiated since 2012 between the 10 ASEAN (Association of South- East Asian Nations) governments and their six FTA partners: Australia, China, India, Japan, New Zealand and South Korea. The 10 ASEAN members are Brunei, Burma (Myanmar), Cambodia, Indonesia, Laos, Malaysia, the Philippines, Singapore, Thailand, Vietnam. It is expected to provide market access for India’s goods and services exports and encourage greater investments and technology into India. It would also facilitate India’s MSMEs to effectively integrate into the regional value and supply chains.

Source) https://www.thehindu.com/news/international/should-india-stay-with-regional- comprehensive-economic-partnership-mooted-by-asean/article28228900.ece

Q.100) Ans) a Exp) A microdot is text or an image substantially reduced in size onto a small disc to prevent detection by unintended recipients. Microdots are normally circular and round one millimetre in diameter but can be made into different shapes and sizes and made from various materials such as polyester or metal. The microdot technology involves spraying thousands of microscopic

ForumIAS Offline Guidance Centre 2nd Floor, IAPL House, 19, Pusa Road, Karol Bagh, New Delhi – 110005 | [email protected]|9821711605 https://t.me/UPSC_PDF www.upscpdf.com

PTS 2020|1|Test 4 - Solutions |ForumIAS

dots onto vehicles or other assets to form a unique identification. Each microdot carries this identification which is registered to the owner, but is not visible to the naked eye. The Ministry of Road Transport and Highways has issued a draft notification to make microdots mandatory in vehicles. The government has envisaged that with microdots becoming a permanent feature in vehicles, identifying them would become easier in case they are stolen.

Source) http://www.newsonair.com/News?title=Road-Transport-and-Highways-Ministry- issues-draft-notification-for-amending-Motor-Vehicles-Rules&id=369319

ForumIAS Offline Guidance Centre 2nd Floor, IAPL House, 19, Pusa Road, Karol Bagh, New Delhi – 110005 | [email protected]|9821711605